Step 2 CK Capstone Sets...

¡Supera tus tareas y exámenes ahora con Quizwiz!

Four days after undergoing open reduction and internal fixation of a fracture of the right femur sustained in a motor vehicle collision, a 47-year-old man continues to have agitation and confusion despite treatment with haloperidol. He has mild hypertension. Other medications include acetaminophen, atenolol, and prophylactic subcutaneous heparin. His temperature is 37.2°C (99°F), pulse is 98/min, respirations are 24/min, and blood pressure is 168/98 mm Hg. During the examination, he is uncooperative and refuses to answer questions. Neurologic examination shows tremulousness and no focal findings. He is oriented to person but not to place or time. A CT scan of the head shows no abnormalities. Which of the following is the most likely cause of these findings? (A) Adverse effect of medication (B) Alcohol withdrawal (C) Fat emboli (D) Sepsis (E) Subdural hematoma

*Answer B is correct.* Agitation, confusion, and tremulousness are consistent with a diagnosis of alcohol withdrawal. Tachycardia and elevated blood pressure are common in withdrawal, but may be abated by antihypertensive medicine (e.g., atenolol). Symptoms of withdrawal can last up to 7 days. Answer A is incorrect. Haloperidol administration would not explain the patient's symptoms. Haloperidol can cause parkinsonism (tremor, rigidity, hypokinesia, and postural instability) because of its anti-dopaminergic properties. Rarely, it can cause neuroleptic malignant syndrome, characterized by hyperthermia, delirium, rigidity, and autonomic instability. Answer C is incorrect. Fat embolism typically occurs after trauma to long bones. Findings include confusion, shortness of breath, hypoxemia, and petechial rash (often associated with anemia and thrombocytopenia). Answer D is incorrect. Sepsis is often associated with fever, altered mentation, and hypotension. Answer E is incorrect. A subdural hematoma should be visible by CT. The patient in this vignette has a normal head CT scan.

A previously healthy 22-year-old college student is brought to the emergency department by her parents 20 minutes after they observed her having a seizure. After the seizure, she was confused and had difficulty thinking of some words. She has had a headache, cough, and fever for 3 days treated with acetaminophen and dextromethorphan. Her temperature is 38.9°C (102°F). Neurologic examination shows diffuse hyperreflexia. On mental status examination, she is confused and has short-term memory deficits. She has difficulty naming objects and makes literal paraphasic errors. An MRI of the brain shows bitemporal hyperintensities. A lumbar puncture is done; cerebrospinal fluid analysis shows an erythrocyte count of 340/mm3, a leukocyte count of 121/mm3, (88% monocytes), and a protein concentration of 78 mg/dL. Which of the following is the most likely diagnosis? (A) Bacterial meningitis (B) Dextromethorphan intoxication (C) Herpes simplex encephalitis (D) HIV encephalopathy (E) Reye syndrome (F) Syphilis

*Answer C is correct.* In a patient with symptoms of encephalitis, the presence of bitemporal hyperintensities on MRI suggests herpes simplex encephalitis. Cerebrospinal fluid findings are consistent with this diagnosis. Answer A is incorrect. Bacterial meningitis would not produce these cerebrospinal fluid findings or cause bitemporal hyperintensities to appear on MRI. Answer B is incorrect. Dextromethorphan intoxication causes hallucinations, gait disturbance, and clinical findings of sympathetic stimulation. Neurobehavioral changes may include euphoria, hallucinations, inappropriate laughing, psychosis with dissociative features, agitation, and coma. Physical findings include tachycardia, dilated pupils, diaphoresis, and "zombie-like" ataxic gait. The patient in this scenario does not have findings of sympathetic stimulation. Answer D is incorrect. HIV encephalopathy is also known as AIDS dementia complex (ADC). Nearly all patients with AIDS develop HIV encephalopathy at some time during the course of the disease. All persons with changes in mental status should be questioned about risk factors for HIV exposure; if any are present, serum testing should be performed. HIV encephalopathy is the most common but by no means the only neurologic manifestation of HIV infection. Although a wide variety of infections and neoplastic brain diseases may occur, the dementia syndrome progresses in a fairly predictable fashion. Forgetfulness, poor concentration, and slowing of thought are early symptoms. Apathy and social withdrawal often appear early and become progressively severe. Psychomotor retardation occurs. Memory impairment and disturbances of higher cortical functioning become apparent as the disease progresses. Delusions, hallucinations, and agitation may occur. The terminal state is usually characterized by "quiet confusion." The patient in this scenario has an acute process manifested by headache, confusion, and seizure, not a chronic process of dementia, suggestive of HIV encephalopathy. Answer E is incorrect. Reye syndrome is a rare but serious condition that causes swelling in the liver and brain. Reye's syndrome most often affects children and teenagers recovering from a viral infection and who may also have a metabolic disorder. Signs and symptoms, such as confusion, seizures and loss of consciousness, require emergency treatment. Early diagnosis and treatment of Reye's syndrome can save a child's life. Aspirin has been linked with Reye's syndrome. Though aspirin is approved for use in children older than age 2, children and teenagers recovering from chickenpox or flu-like symptoms should never take aspirin. In Reye's syndrome, a child's blood sugar level typically drops while the levels of ammonia and acidity in his or her blood rise. At the same time, the liver may swell and develop fatty deposits. Swelling may also occur in the brain, which can cause seizures, convulsions or loss of consciousness. The signs and symptoms of Reye's syndrome typically appear about three to five days after a viral infection, such as the flu (influenza) or chickenpox, or an upper respiratory infection, such as a cold. Reye's syndrome is not associated with the cerebrospinal fluid findings or bitemporal changes seen in this patient. Answer F is incorrect. Neurosyphilis is simply the occurrence of neurologic complications due to infection with T pallidum. It may occur during early or late syphilis. The spectrum of neurosyphilis is broad. The most common form of neurosyphilis currently diagnosed is asymptomatic neurosyphilis. Individuals with this form of neurosyphilis come to medical attention because of serological evidence of syphilis in the absence of neurologic sequelae. Examination of the cerebrospinal fluid reveals evidence of neurosyphilis; these patients are at risk for developing symptomatic disease. Among the symptomatic disorders of neurosyphilis, the earliest manifestation is syphilitic meningitis, which typically occurs within the first 12 months of infection and may accompany features of secondary syphilis (maculopapular rash affecting the palms and soles). Headaches, meningismus, cranial nerve palsies (chiefly, in descending order of frequency, VII, VIII, VI, and II), hearing loss, tinnitus, and vertigo may be observed in isolation or combination in upwards of 40% of patients with secondary syphilis. Impaired vision secondary to chorioretinitis, retinitis, optic neuropathy, optic chiasmal, or optic tract disease has also been reported. The symptoms of syphilitic meningitis include headache, photophobia, and a stiff neck. Encephalopathic features resulting from vascular compromise or increased intracranial pressure may be observed. These include confusion, lethargy, seizures, aphasia, and hemiplegia. Intractable seizures may, on rare occasions, be the initial manifestation of neurosyphilis. Acute sensorineural hearing loss and acute optic neuritis may occur in association with syphilitic meningitis or independently. Neurosyphilis is no associated with bitemporal hyperintensities on MRI, as were seen in this patient.

A 75-year-old man presents with problems walking that have developed over the previous 2 years, consisting of slow gait, imbalance (especially on turning), short stride length, and gait initiation failure. He reports urinary frequency, occasional urge incontinence, and some memory loss. On examination, his symptoms are symmetric and much more prominent in the lower half of the body, with relative sparing of hand function, and normal facial expressiveness. He has previously been diagnosed with Parkinson disease; however, therapy with levodopa has not improved his symptoms. These findings are most consistent with which of the following conditions? A. Alzheimer disease B. Frontotemporal dementia C. Lewy body disease D. Normal-pressure hydrocephalus E. Parkinson disease F. Vascular dementia

*Answer D is correct.* Normal-pressure hydrocephalus is associated with dementia, gait instability, and urinary incontinence.

A 20-year-old woman with no significant past medical history presents with lower back pain and bilateral foot and hand tingling. Her symptoms rapidly progress over 4 days to include lower extremity weakness to the point that she is unable to mobilize her lower extremities. She reports coryzal symptoms 2 weeks ago. On examination, she has 0/5 power in her lower extremity with areflexia, but despite the paresthesias she does not have sensory deficits. Her aminotransferases are elevated, and CSF examination reveals mildly elevated protein with no cells and normal glucose. Which of the following is the most likely diagnosis? A. Amyotrophic lateral sclerosis B. Botulism C. Guillain-Barré syndrome D. Multiple sclerosis E. Myasthenia gravis

Answer C is correct. Guillain-Barré syndrome. Guillain-Barré syndrome is an acute inflammatory demyelinating polyneuropathy (AIDP), an autoimmune disorder affecting the peripehral nervous system, usually triggered by an acute infectious process. Typically, the patient experiences ascending paralysis, in which leg weakness spreads to the upper limbs and face. There is complete loss of deep tendon reflexes. With prompt treatment, the majority of patients recover. Death may result, however, from respiratory or autonomic nervous system dysfunction. Amyotrophic lateral sclerosis (ALS). ALS is a disease that affects upper and lower motor neurons. Most patients present with spasticity and hyperreflexia (upper motor neuron signs) and progressive muscle weakness and atrophy (lower motor neuron signs). Cognitive function, bowel and bladder function, and eye movements are usually preserved. The disease is relentless and progressive: Affected individuals lose control of voluntary movements, and respiratory paralysis ensues. Sensory and autonomic nerves are usually spared. ALS may be familial or acquired. The two forms are clinically and pathologically indistinguishable. Botulism. Botulism is a rare, toxin-mediated, paralytic disease. The toxin is produced by the bacterium Clostridium botulinum. The disease may be acquired by bacterial colonization of the digestive tract, by direct ingestion of toxin, or by wound contamination. Paralysis typically starts with the cranial nerves and spreads to the limbs. The patient may present with double vision, drooping of the eyelids, loss of facial expression, or difficulty with swallowing or talking. The weakness then spreads to the arms (starting in the shoulders and proceeding to the forearms) and legs (starting with the thighs and proceeding to the feet). The autonomic nervous sytem may be affected, causing dry mouth and throat, postural hypotension, and constipation. Nausea and vomiting may occur, and, in severe forms, respiratory paralysis ensues. Multiple sclerosis. Multiple sclerosis is an inflammatory disease in which the myelin sheaths around the axons of the brain and spinal cord are damaged, leading to demyelination and scarring. (Myelin is a substance that provides "electrical insulation." It forms a layer, the myelin sheath, which is usually around only the axon of a neuron. Schwann cells supply the myelin for peripheral neurons, whereas oligodendrocytes myelinate the axons of the central nervous system.) Multiple sclerosis usually begins in young adults. Women are more commonly affected than men. Patients can have almost any neurological finding, including fatigue, visual changes, altered sensation, pain, muscle weakness, difficulty with movement or balance, problems with speech or swallowing, and bowel or bladder dysfunction. Cognitive and mood changes may occur. Uhthoff's phenomenon (exacerbation of symptoms with exposure to heat) and Lhermitte's sign (an electrical sensation running down the back when bending the neck) are characteristic but not specific for multiple sclerosis. Symptoms may progress gradually or alternate between relapse and remission. Myasthenia gravis. Myasthenia gravis is an autoimmune neuromuscular disease that causes fluctuating muscle weakness and fatigability. Weakness is caused by circulating antibodies that block acetylcholine receptors at the post-synaptic neuromuscular junction. The hallmark of myasthenia gravis is fatigability: Muscles become weaker with activity and stronger with rest. Especially susceptible are the muscles that control eye and eyelid movements, facial expression, chewing, talking, and swallowing. Presenting symptoms include ptosis, diplopia, dysarthria, dysphagia, and change in facial expression. Neck and limb movements may be affected, gait may be unsteady, and breathing may be impaired.

A 47-year-old woman with end-stage renal disease comes to the physician because of increased shortness of breath since her last hemodialysis 2 days ago. Her pulse is 88/min and regular, respirations are 26/min and slightly labored, and blood pressure is 176/110 mm Hg. Examination shows jugular venous distention and pitting edema below the knees. Diffuse crackles are heard. Cardiac examination shows no murmurs, rubs, or gallops. Laboratory studies show: Serum Na+ 138 mEq/L Cl− 100 mEq/L Arterial blood gas analysis on room air: pH 7.30 PCO2 28 mm Hg PO2 88 mm Hg HCO3− 14 mEq/L Which of the following is the most likely acid-base status of this patient? A. Metabolic acidosis, respiratory compensation B. Metabolic acidosis, uncompensated C. Metabolic alkalosis, respiratory compensation D. Metabolic alkalosis, uncompensated E. Respiratory acidosis, renal compensation F. Respiratory acidosis, uncompensated G. Respiratory alkalosis, renal compensation H. Respiratory alkalosis, uncompensated I. Normal acid-base balance

Answer A is correct. A normal serum pH is 7.4 + 0.03. This patient's pH is 7.3, indicating acidosis. A normal serum HCO3− level is 24 mEq/L. This patient's serum HCO3− level is 14 mEq/L, indicating a metabolic acidosis. A normal anion gap is 12 + 2 mmol/L. This patient's anion gap is 24 mmol/L, indicating an increased anion gap metabolic acidosis. If the metabolic acidosis were uncompensated, one would expect a near-normal pCO2 (40 mm Hg). The predicted pCO2 in a fully compensated patient is calculated as follows: Expected pCO2 = 1.5 (Actual [HCO3]) + 8 mmHg. Given a HCO3− level of 14 mEq/L, the predicted pCO2 would be 29 mm Hg, which approximates the patient's pCO2 of 28 mm Hg. This patient has a compensated increased-anion-gap metabolic acidosis.

A 67-year-old man with chronic cough comes to the emergency department after he coughed up 1 cup of bright red blood this morning. He has a 3-week history of increasingly severe cough productive of whitish yellow sputum and a 1-week history of fever. Over the past 3 days, the sputum has contained small amounts of blood. He has not had chest pain. He has type 2 diabetes mellitus and hypertension. He has a 40-year history of alcoholism and drinks eight alcoholic beverages daily. He had tuberculosis 15 years ago treated for 1 year with isoniazid and rifampin. His temperature is 37.8°C (100°F), pulse is 108/min, respirations are 22/min, and blood pressure is 130/84 mm Hg. Scattered rhonchi are heard that move with coughing; there are no wheezes. Cardiac examination shows a normal rhythm and no murmurs or gallops. There is trace pedal edema bilaterally. An x-ray of the chest shows a single cavitary lesion. A CT scan of the chest shows a soft tissue mass within the cavity. Which of the following is the most likely diagnosis? (A) Aspergilloma (B) Bacterial lung abscess (C) Bronchogenic carcinoma (D) Histoplasmosis (E) Tuberculosis

Answer A is correct. Aspergillomas usually occur in pre-formed cavities, especially cavities produced by tuberculosis. Hemoptysis is the most common presenting symptom. CT may show a mass within a cavity.

A previously healthy 14-year-old girl comes to the physician because of a 2-month history of exercise-induced cough and nasal congestion. She plays field hockey and has noticed she coughs when running up and down the field. The cough is nonproductive and resolves with rest. She has not had chest pain or palpitations. She takes no medications and does not smoke. Her sister has asthma. The patient appears well. Her pulse is 68/min, respirations are 16/min, and blood pressure is 100/75 mm Hg. Pulse oximetry on room air shows an oxygen saturation of 99%. Cardiopulmonary examination shows no abnormalities. An x-ray of the chest shows no abnormalities. Spirometry shows an FEV1:FVC ratio of 90% and an FEV1 of 90% of predicted. Which of the following is the most likely diagnosis? (A) Asthma (B) Chronic bronchitis (C) Gastroesophageal reflux disease (D) Postnasal drip syndrome (E) Variable endothoracic upper airway obstruction

Answer A is correct. Asthma is one of the most common causes of chronic cough. In cough-variant asthma, cough is the only symptom. Cough that is precipitated by exercise and relieved by rest is highly suggestive of asthma. The chest x-ray is typically normal in patients with asthma. When the patient is not having an attack, the lung exam and spirometry are normal as well. Patients with asthma often have a family history of asthma, as does the patient in this vignette. Answer B is incorrect. Chronic bronchitis is defined as a persistent cough resulting in sputum production for more than 3 months in each of the last 3 years. Answer C is incorrect. Gastroesophageal reflux disease (GERD), a common cause of cough, is rare in young people. Its prevalence increases with age. GERD-induced cough is usually worse at night and not associated with exercise. Answer D is incorrect. Postnasal drip syndrome can cause cough. It is usually worse at night and the patient common feels mucus going down the throat. It is not usually triggered by exercise. Answer E is incorrect. Upper airway obstruction often causes stridor; lower airway obstruction often causes wheezing.

An asymptomatic 72-year-old man comes to the physician for a routine examination. He has had several episodes of gout over the past 6 years. His only medication is allopurinol. During a visit two months ago, his blood pressure was 172/96 mm Hg. During a visit one month ago for an uncomplicated upper respiratory tract infection, his blood pressure was 170/94 mm Hg. Today, his pulse is 76/min, and blood pressure is 166/98 mm Hg. Examination shows no other abnormalities. Laboratory findings are within the reference range. An ECG shows borderline left ventricular hypertrophy. Which of the following is the most appropriate next step in management? (A) Repeat blood pressure measurement in 3 months (B) Order renal Doppler ultrasonography (C) Add lisinopril to the regimen (D) Add hydrochlorothiazide to the regimen (E) Discontinue allopurinol therapy

Answer C is correct. The patient has two blood pressure readings in the hypertensive range (>140/90 mm Hg), indicating that he needs treatment for hypertension. The need is especially pressing because the patient has target organ damage, as manifested by left ventricular hypertrophy. A diuretic like hydrochlorothiazide would be a good choice if the patient didn't have gout, but diuretics are relatively contraindicated in patients with gout because of the propensity for hyperuricemia. An ACE-inhibitor like lisinopril would be a good alternative. Answer A is incorrect. The diagnosis of hypertension is confirmed by two blood pressure readings taken at different times and found to be in the hypertensive range (>140/90 mm Hg). The presence of left ventricular hypertrophy lowers the threshold for treatment and lowers the target blood pressure goal (<130/80 mm Hg). Answer B is incorrect. Renal Doppler ultrasonography is used to diagnose renal artery stenosis, a form of secondary hypertension. There are many types of secondary hypertension: renal artery stenosis, renal parenchymal disease, Cushing disease, hyperaldosteronism, pheochromocytoma, hypo- and hyper-thyroidism, obstructive sleep apnea, etc. A secondary cause of hypertension should be considered when the clinical presentation is suggestive; however, searching for a secondary cause is not cost-effective in all patients with hypertension. The clinical presentation should guide the search for a secondary cause. In this patient, there is no reason to suspect renal artery stenosis. Answer D is incorrect. Hydrochlorothiazide is usually an excellent first choice for treating hypertension, but it should not be used in patients with gout. Answer E is incorrect. There is no reason to discontinue allopurinol.

A 42-year-old woman comes to the physician because of an 8-week history of intermittent nausea and abdominal pain that occurs 20 to 30 minutes after eating. The pain extends from the epigastrium to the right upper quadrant and is sometimes felt in the right scapula; it lasts about 30 minutes and is not relieved by antacids. The last episode occurred after she ate a hamburger and French fries. She has not had vomiting. She is currently asymptomatic. She is 165 cm (5 ft 5 in) tall and weighs 104 kg (230 lb); BMI is 38 kg/m2. Examination shows no other abnormalities. Which of the following is the most appropriate next step in management? (A) Abdominal ultrasonography of the right upper quadrant (B) Upper gastrointestinal series with small bowel follow-through (C) CT scan of the abdomen (D) Endoscopic retrograde cholangiopancreatography (E) Elective cholecystectomy (F) Immediate cholecystectomy

Answer A is correct. Biliary colic is typically a steady ache in the epigastrium or right upper quadrant, of sudden onset, reaching a plateau of intensity over a few minutes, which subsides gradually over 30 minutes to several hours. Referred pain may be felt at the tip of the scapula or right shoulder. Nausea and vomiting may occur. Gallstones can be best demonstrated by transabdominal ultrasonography (sensitivity and specificity, >95%), which has become the initial test to evaluate cholelithiasis.

A 37-year-old woman with AIDS comes to the physician because of a 1-month history of progressive diarrhea and a 1.8-kg (4-lb) weight loss. During the past week, she has had six large watery stools daily. She is currently receiving triple antiretroviral therapy. She is employed as a flight attendant and works regularly on domestic flights throughout the USA. She also flies to Asia at least once monthly. She is 163 cm (5 ft 4 in) tall and weighs 59 kg (130 lb); BMI is 22 kg/m2. Her temperature is 37°C (98.6°F), pulse is 88/min, and blood pressure is 112/64 mm Hg. The abdomen is scaphoid. The remainder of the examination shows no abnormalities. Her CD4+ T-lymphocyte count is 400/mm3 (normal ≥ 500). Which of the following is the most likely causal organism? (A) Cryptosporidium parvum (B) Cytomegalovirus (C) Mycobacterium avium-intracellulare complex (D) Salmonella enteritidis (E) Strongyloides stercoralis

Answer A is correct. Cryptosporidium parvum is a common cause of large-volume watery diarrhea in patients with AIDS. Answer B is incorrect. Cytomegalovirus causes colitis (small bowel movements with blood or mucus) in patients with AIDS. It also causes esophageal ulcers and hepatitis. Answer C is incorrect. Mycobacterium avium-intracellulare complex causes fever, abdominal pain, and lymphadenopathy in patients with AIDS. Answer D is incorrect. Salmonella enteritidis causes colitis (small bowel movements with blood or mucus) in patients with AIDS. Answer E is incorrect. Strongyloides stercoralis is a common cause of eosinophilia in returning long-term travelers and immigrants, particularly those from Southeast Asia. Infection occurs when the parasite penetrates the skin, migrates to the lungs, and then is swallowed. Infection can persist for years; many men who served in the Pacific theater during World War II or in Vietnam still harbor infections. S. stercoralis is unique in its ability to complete its life cycle in the host and produce persistent infection. Although usually asymptomatic, infection can cause diarrhea, abdominal pain, and malabsorption. This helminth can cause life-threatening disseminated infection in individuals who are immunosuppressed from cancer chemotherapy, corticosteroids, or HIV infection. Diagnosis may be made by stool examination, but this is not a very sensitive technique. Treatment with thiabendazole, 25 mg/kg (maximum 3 g/day) orally twice daily for 2 days, is curative in more than 90% of immunocompetent hosts. Ivermectin (200 mcg/kg/day) may be more effective than thiabendazole and is now the preferred drug for treatment of strongyloidosis, especially for immunocompromised hosts.

A full-term newborn is brought to the emergency department at 5 days of age because of abnormal extremity movements for 6 hours. The newborn was delivered at home to a 21-year-old primigravid woman who received no prenatal care. The infant is alert, afebrile, hypertonic, and hyperreflexic. Complete blood count, serum electrolyte concentrations, and cerebrospinal fluid analysis are within normal limits. Which of the following is the most likely pathogen? (A) Clostridium tetani (B) Cytomegalovirus (C) Escherichia coli (D) Group B streptococcus (E) Herpes simplex virus (F) Listeria monocytogenes (G) Parvovirus B19 (H) Rubella virus (I) Toxoplasma gondii (J) Treponema pallidum

Answer A is correct. Diffuse hypertonicity and hyperreflexia in an alert infant suggests tetanus.

A 67-year-old man is brought to the emergency department because of a 1-week history of nausea, generalized weakness, and malaise. He has congestive heart failure, hypertension, and coronary artery disease. Current medications include lisinopril, digoxin, isosorbide, spironolactone, and metoprolol. His temperature is 37.2°C (99°F), pulse is 88/min, and blood pressure is 140/90 mm Hg. Examination shows a soft abdomen. There is 2+ edema in the lower extremities. Laboratory studies show: Hematocrit 36% Leukocyte count 10,000/mm3 Na+ 140 mEq/L K+ 7.3 mEq/L HCO3− 8 mEq/L Urea nitrogen 40 mg/dL Creatinine 1.8 mg/dL AST 20 U/L Urinalysis shows no abnormalities. Which of the following is the most likely explanation for this patient's hyperkalemia? (A) Adverse effect of medications (B) Laboratory error (C) Metabolic acidosis (D) Renal failure (E) Rhabdomyolysis

Answer A is correct. Drugs are a common cause of hyperkalemia and hypokalemia. This patient is taking spironolactone, a potassium-sparing diuretic. Spironolactone inhibits aldosterone, resulting in volume depletion, hyperkalemia, and acidosis. All of these findings are present in this patient.

A 19-year-old woman noticed a mass in her left breast 2 weeks ago while doing monthly breast self-examination. Her mother died of metastatic breast cancer at the age of 40 years. Examination shows large dense breasts; a 2-cm, firm, mobile mass is palpated in the upper outer quadrant of the left breast. There are no changes in the skin or nipple, and there is no palpable axillary adenopathy. Which of the following is the most likely diagnosis? (A) Fibroadenoma (B) Fibrocystic changes of the breast (C) Infiltrating ductal carcinoma (D) Intraductal papilloma (E) Lobular carcinoma

Answer A is correct. Fibroadenomas are much more common than breast cancer in women under the age of 35. A firm, mobile, 2-cm mass in a young woman is most likely to be a fibroadenoma, the most common benign breast neoplasm in women. Answer B is incorrect. Fibrocystic change is unlikely to present as a firm, mobile, 2-cm mass. Answer C is incorrect. Breast cancer would be an unlikely diagnosis in a 19-year-old woman. Answer D is incorrect. Intraductal papillomas are usually 1 to 3 mm in diameter and typically present with watery, serous, or bloody nipple discharge. Answer E is incorrect. Breast cancer would be an unlikely diagnosis in a 19-year-old woman.

A 45-year-old woman has a 2-week history of increased anxiety, abdominal discomfort, irritability, and difficulty concentrating; she was robbed at knifepoint in a parking lot 3 weeks ago. She takes levothyroxine for hypothyroidism and uses an over-the-counter inhaler as needed for exercise-induced asthma. Her blood pressure is 140/80 mm Hg, and pulse is 100/min. Examination shows dry skin and hair. She is cooperative but appears anxious, glancing around quickly when a loud noise is heard outside the office. Leukocyte count is 12,000/mm3, and serum thyroid-stimulating hormone concentration is 5.0 μU/mL. An ECG shows sinus tachycardia. Which of the following is the most likely diagnosis? (A) Acute stress disorder (B) Agoraphobia (C) Generalized anxiety disorder (D) Hypothyroidism (E) Panic disorder

Answer A is correct. Individuals exposed to severely stressful events (typically involving the actual or threatened loss of life or limb) may experience any of a wide variety of psychiatric sequelae. If the sequelae include symptoms of re-experiencing of the traumatic event (e.g., intrusive memories, nightmares, flashbacks), avoidance phenomena (e.g., avoiding reminders of the event, sense of foreshortened future), and hyperarousal symptoms (e.g., irritability, exaggerated startle response), the disorder is termed acute stress disorder (if the duration is up to 1 month). In the vignette above, the patient has hyperarousal less than 1 month after a life-threatening event. The most likely diagnosis is acute stress disorder. Answer B is incorrect. Patients with agoraphobia have anxiety about being in places from which escape might be difficult or embarrassing, e.g., being alone, in crowds, in tunnels, or on bridges. Answer C is incorrect. The patient in the vignette does not have generalized anxiety, but rather, a specific anxiety related to a life-threatening event. Answer D is incorrect. This patient has a mildly elevated TSH and may be hypothyroid. Hypothyroidism can cause dry hair and dry skin, but it would not cause the hyperadrenergic symptoms experienced by this patient. Hyperthyroidism, by contrast, would. Answer E is incorrect. A panic attack is a transient episode of crescendo anxiety, catastrophic thoughts (e.g., fears of dying, going insane, or losing self-control), and somatic symptoms. Panic disorder manifests with recurrent panic attacks of which at least some are unexpected and unpredictable.

A 37-year-old woman is brought to the emergency department because of intermittent chest pain for 3 days. The pain is worse with inspiration, and she feels she cannot take deep breaths. She has not had shortness of breath, palpitations, or nausea. She had an upper respiratory tract infection 10 days ago and took an over-the-counter cough suppressant and decongestant and acetaminophen. Her temperature is 37.2°C (98.9°F), pulse is 90/min, and blood pressure is 122/70 mm Hg. The lungs are clear to auscultation. S1 and S2 are normal. A rub is heard during systole. There is no peripheral edema. An ECG shows normal sinus rhythm and diffuse, upwardly concave ST-segment elevation and PR-segment depression in leads II, III, and aVF. Which of the following is the most likely diagnosis? (A) Acute pericarditis (B) Aortic dissection (C) Gastroesophageal reflux disease (D) Myocardial infarction (E) Peptic ulcer disease (F) Pulmonary embolism (G) Unstable angina pectoris

Answer A is correct. Pain that is preceded by an upper respiratory tract infection, worsened by inspiration, and associated with a pericardial friction rub and characteristic ECG changes (PR-segment depression and upwardly concave ST-segment elevation) suggests a diagnosis of acute pericarditis. Answer B is incorrect. Aortic dissection causes excruciating, "tearing" chest pain that radiates to the back or interscapular region. It is sudden in onset and unrelenting, and usually occurs in the setting of hypertension of other predisposing condition (e.g., Marfan syndrome). The patient may have asymmetric blood pressures in the upper extremities and a heart murmur due to aortic insufficiency. Answer C is incorrect. Gastroesophageal reflux causes burning epigastric or substernal chest pain that may radiate to the neck. Symptoms are worse after a meal and with recumbency. Antacids relieve the discomfort. Answer D is incorrect. Myocardial infarction usually causes severe and prolonged substernal chest pain associated with shortness of breath, nausea, and diaphoresis. Electrocardiographic changes may include ST-segment elevation or ST-segment depression with T-wave inversion. Answer E is incorrect. Peptic ulcers may cause burning or aching in the epigastric or substernal area. Symptoms are relieved by food and antacids. Answer F is incorrect. Pulmonary embolism can cause sudden onset chest pain that is aggravated by breathing and associated with shortness of breath and tachycardia. Answer G is incorrect. Unstable angina causes substernal chest pain, often described as a pressure, squeezing, tightness, or heaviness. The pain may radiate to the neck, jaw, or shoulder, and may be associated with electrocardiographic changes.

During the past month, a 37-year-old woman has had epigastric pain 2 to 3 hours after eating and at night; she has a feeling of fullness and bloating even when she eats small amounts. For 2 days, she has been unable to keep any food "down" and has had repetitive vomiting between meals. Six months ago, she was diagnosed with a peptic ulcer and was treated with a proton pump inhibitor and antibiotics. After 2 weeks of treatment, her symptoms were alleviated, and she discontinued the medication due to the quantity of pills she had to take. Placement of a Foley catheter yields no urine. This patient is most likely to have which of the following electrolyte profiles? Na+ K+ Cl− HCO3 (mEq/L) (mEq/L) (mEq/L) (mEq/L) (A) 130 2.8 88 32 (B) 130 4.2 100 24 (C) 130 4.4 100 14 (D) 148 2.3 96 24 (E) 148 4.8 110 24

Answer A is correct. Prolonged vomiting is associated with hypokalemia and metabolic alkalosis. Hyponatremia also occurs because of sodium loss and appropriate activation of ADH related to volume contraction. Answer B is incorrect. "Plain" hyponatremia occurs in SIADH (syndrome of inappropriate antidiuretic hormone secretion). Answer C is incorrect. Increased-anion-gap metabolic acidosis with hyponatremia occurs in diabetic ketoacidosis. Answer D is incorrect. Hypernatremia is a sign of dehydration. Hypernatremia with hypokalemia occurs in diabetes insipidus. Answer E is incorrect. "Plain" hypernatremia occurs in simple dehydration.

A 32-year-old woman, gravida 3, para 2, at 41 weeks' gestation is admitted to the hospital in active labor. Pregnancy has been complicated by mild asthma treated with inhaled bronchodilators. At the beginning of the second stage of labor, the cervix is 100% effaced and 10 cm dilated; the vertex is at -1 station. The fetal heart rate is reactive with no decelerations. After 10 minutes of pushing, there is a prolonged deceleration to 60/min. The patient has the acute onset of shortness of breath, rapidly develops cyanosis, and becomes unresponsive. Her pulse and blood pressure cannot be detected. Immediate resuscitation is started. Five minutes later, there is bleeding from the nose, mouth, and intravenous sites. Which of the following is the most likely diagnosis? (A) Amniotic fluid embolism (B) Intracerebral hemorrhage (C) Myocardial infarction (D) Status asthmaticus (E) Toxic shock syndrome

Answer A is correct. Sudden onset shortness of breath, hypoxemia, and cardiovascular collapse during labor suggests amniotic fluid embolism. Advanced maternal age and multiparity are predisposing factors. Bleeding from the mouth, nose, and intravenous sites is due to a consumptive coagulopathy with thrombocytopenia and subsequent fibrinolysis.

A 6-year-old boy is brought to the emergency department 2 hours after injuring his arm when he fell out of a tree. His mother says that he is extremely active and likes to climb. During the past year, he fractured his right tibia after falling off a trampoline and sustained a concussion after falling off his bicycle. She says that his teachers reprimand him frequently for running wildly in the classroom, talking excessively, and getting out of his seat; he often forgets to turn in his homework. His parents are currently divorcing. His father has a history of illicit drug use. The patient is at the 50th percentile for height and weight. His pulse is 80/min, and blood pressure is 100/80 mm Hg. Physical examination shows a dislocated left shoulder, healing abrasions over the elbows, and ecchymoses in various stages of healing over the knees. Mental status examination shows a neutral affect. He says that he likes to run and climb trees. Which of the following is the most likely explanation for these findings? (A) Attention-deficit/hyperactivity disorder (B) Conduct disorder (C) Learning disorder (D) Seizure disorder (E) Age-appropriate behavior

Answer A is correct. The boy in the vignette has signs of hyperactivity that 1) began before age seven, 2) have lasted more than a year, 3) have occurred in at least two settings, 4) are inconsistent with developmental level, 5) are impairing school performance, and 6) cannot be attributed to another disorder. This boy meets the criteria for ADHD. Answer B is incorrect. A conduct disorder is a psychological disorder diagnosed in childhood or adolescence that presents itself through a repetitive and persistent pattern of behavior in which the basic rights of others or major age-appropriate norms are violated. These behaviors are often referred to as "antisocial behaviors." Indeed, the disorder is often seen as the precursor to antisocial personality disorder. Answer C is incorrect. A learning disorder is a disorder found in children of normal intelligence who have difficulties in learning specific skills. Answer D is incorrect. The boy's symptoms are unlikely to be caused by seizure activity. Answer E is incorrect. This is not age-appropriate behavior.

A 27-year-old woman comes to the physician for a follow-up examination. She has a 10-week history of persistent nonproductive cough that is worse at night and a 1-month history of a hoarse voice. She otherwise feels well. She has not had loss of appetite or change in exercise tolerance. She has never smoked. Empiric treatment with an oral decongestant and albuterol inhaler has not improved her symptoms. She has no history of serious illness. Her temperature is 37°C (98.6°F), pulse is 68/min, respirations are 12/min, and blood pressure is 110/76 mm Hg. Cardiopulmonary examination and an x-ray of the chest show no abnormalities. Her FEV1 is normal. Which of the following is the most appropriate next step in management? (A) 24-Hour monitoring of esophageal pH (B) Echocardiography (C) CT scan of the chest (D) Inhaled corticosteroid therapy (E) Nitroglycerin therapy

Answer A is correct. The classic symptom of gastroesophageal reflux disease is heartburn; however, many patients with GERD present atypically. GERD is a common condition that should be suspected in patients with unexplained chest pain, cough, wheezing, hoarseness, chronic sore throat, or globus sensation. Typically symptoms are worse at night. Evaluation may include 24-Hour monitoring of esophageal pH or empiric administration of acid antisecretory drugs.

A 15-year-old boy is brought to the physician because of fatigue since starting his freshman year of high school 3 months ago. He often falls asleep during class. He urinates four to five times nightly and often has difficulty falling asleep again. He has no history of serious illness and takes no medications. He is at the 20th percentile for height and above the 95th percentile for weight and BMI. Vital signs are within normal limits. Examination shows a velvety, hyperpigmented, macular rash over the neck and axillae. The remainder of the examination shows no abnormalities. Results of a complete blood count and serum electrolyte concentrations show no abnormalities. Additional laboratory studies show: Serum glucose 134 mg/dL Urine pH 5.5 Specific gravity 1.028 Glucose 1+ Ketones negative In addition to dietary counseling, which of the following is the most appropriate initial treatment? (A) Exercise program (B) Increased fluid intake (C) Cyclosporine therapy (D) Insulin therapy (E) Oral hypoglycemic agent

Answer A is correct. The patient has diabetes mellitus, as indicated by glycosuria and an elevated serum glucose level. His symptoms are mild and subacute, typical of type 2 diabetes. (Patients with type 1 diabetes present acutely and are more symptomatic.) The first step in treating type 2 diabetes is lifestyle modification: diet, exercise, and weight loss. Type 2 diabetes mellitus is increasingly diagnosed in children and young adults, due poor diet, sedentary lifestyle, and obesity.

A 2-week-old newborn is brought to the physician because his lips have turned blue on three occasions during feeding; he also sweats during feeding. He was born at 38 weeks' gestation and weighed 2466 g (5 lb 7 oz); he currently weighs 2778 g (6 lb 2 oz). His temperature is 37.8°C (100°F), pulse is 170/min, respirations are 44/min, and blood pressure is 75/45 mm Hg. A grade 3/6 harsh systolic ejection murmur is heard at the left upper sternal border. An x-ray of the chest shows a small boot-shaped heart and decreased pulmonary vascular markings. Which of the following is the most likely diagnosis? (A) Anomalous coronary vessels (B) Atrial septal defect (C) Endocardial fibroelastosis (D) Tetralogy of Fallot (E) Total anomalous pulmonary venous return

Answer D is correct. Hypercyanotic attacks in a patient with a boot-shaped heart and a systolic murmur of pulmonary stenosis suggest Tetralogy of Fallot.

A 65-year-old man who is quadriplegic as a result of multiple sclerosis is hospitalized for treatment of left lower lobe pneumonia. His temperature is 38.1°C (100.5°F), pulse is 95/min, respirations are 12/min, and blood pressure is 120/80 mm Hg. He appears malnourished. Rhonchi are heard at the left lower lobe of the lung on auscultation. Examination of the heart, lymph nodes, abdomen, and extremities shows no abnormalities. There is a 1-cm area of erythema over the sacrum with intact skin and no induration. Neurologic examination shows quadriparesis. Test of the stool for occult blood is negative. Which of the following is the most effective intervention for this patient's skin lesion? (A) Frequent turning (B) Use of wet to dry dressings (C) Whirlpool therapy (D) Broad-spectrum antibiotic therapy (E) Surgical debridement

Answer A is correct. The patient in the vignette has a stage I pressure "ulcer" (erythema of intact skin). Frequent turning is indicated. Answer B is incorrect. Wet-to-dry dressings are beneficial in patients with stage II and above pressure ulcers. Dressings are needed in stage I. Answer C is incorrect. Whirlpool therapy is beneficial for pressure ulcers with copious exudate. It is not needed for stage I ulcers. Answer D is incorrect. Broad-spectrum antibiotic coverage is recommended for patients with infected ulcers. Antibiotics are not beneficial in patients with stage I ulcers. Answer E is incorrect. Surgical debridement may be beneficial for ulcers with purulent or necrotic tissue. Debridement is not indicated for stage I ulcers.

A 4-year-old boy is brought to the physician because of temperatures to 39.4°C (102.9°F) for 8 days. Examination shows anterior cervical lymphadenopathy, nonexudative conjunctivitis bilaterally, a strawberry tongue, an erythematous truncal rash, and edema of the hands and feet. Which of the following is the most appropriate pharmacotherapy to prevent complications of this illness? (A) Intravenous immune globulin (B) Intravenous penicillin (C) Intravenous prednisone (D) Oral isoniazid (E) Oral rifampin

Answer A is correct. The presence of fever, cervical lymphadenopathy, nonexudative conjunctivitis bilaterally, strawberry tongue, erythematous truncal rash, and edema of the hands and feet suggest a diagnosis of Kawasaki disease. Intravenous immune globulin is the treatment of choice. Answer B is incorrect. Penicillin is effective against Streptococcus Group A, the cause of scarlet fever. The presence of conjunctivitis in this patient, however, argues against the diagnosis of scarlet fever. Classic findings of scarlet fever include fever, sore throat, "strawberry" tongue, and rash. Rash is the most striking sign of scarlet fever. It looks like a sun burn at first. The rash usually appears on the neck and face, often leaving a clear unaffected area around the mouth. It then spreads to the chest and back and finally to the rest of the body. In the body creases, especially around the axillae and elbows, the rash forms the classic red streaks known as Pastia lines. Answer C is incorrect. Intravenous immune globulin, not prednisone, is recommended for treatment of Kawasaki disease. Answer D and E are incorrect. Isoniazid and rifampin are used to treat tuberculosis. Tuberculosis can present with cervical lymphadenopathy (scrofula), but the findings in this case suggest Kawasaki disease.

A previously healthy 19-year-old college student comes to student health services 24 hours after the onset of headache, stiff neck, and sensitivity to light. She does not recall any sick contacts. She had chickenpox at the age of 7 years. Her most recent examination 1 year ago included PPD skin testing and showed no abnormalities. She takes a daily multivitamin and an herbal weight-loss preparation. She received all appropriate immunizations during childhood but has not received any since then. She does not smoke, drink alcohol, or use illicit drugs. There is no family history of serious illness. She appears lethargic. Her temperature is 39.1°C (102.4°F), pulse is 112/min, respirations are 20/min, and blood pressure is 100/68 mm Hg. Examination shows diffuse petechiae. Kernig and Brudzinski signs are present. The remainder of the examination shows no abnormalities. A lumbar puncture is performed. Cerebrospinal fluid (CSF) analysis shows numerous segmented neutrophils and a decreased glucose concentration. A Gram stain of the CSF shows gram-negative cocci. Which of the following is the most appropriate pharmacotherapy? (A) Ceftriaxone (B) Clindamycin (C) Erythromycin (D) Metronidazole (E) Vancomycin

Answer A is correct. The presence of fever, headache, stiff neck, petechiae, positive Kernig and Brudzinski signs, and CSF with neutrophils, low glucose, and gram-negative diplococci suggests meningococcal meningitis. A third-generation cephalosporin, such as ceftriaxone, is the treatment of choice.

A 62-year-old woman comes to the physician because of a 3-day history of a rash over her face and hands that has not improved with the use of skin moisturizers, antibiotic ointments, or corticosteroid cream. She has a 1-month history of progressive weakness. She has difficulty rising from a chair or reaching over her head. She has not had any pain. Vital signs are within normal limits. A photograph of the hands is shown. Muscle strength is 3/5 in the proximal upper and lower extremities. Which of the following is the most likely diagnosis? (A) Dermatomyositis (B) Myasthenia gravis (C) Psoriasis (D) Systemic lupus erythematosus (E) Systemic sclerosis (scleroderma)

Answer A is correct. The presence of proximal muscle weakness in the upper and lower extremities, along with Gottron's papules (seen in the image), suggests a diagnosis of dermatomyositis.

A 32-year-old man who is a jackhammer operator comes to the physician because of pain and swelling of his right arm for 3 days. The symptoms are moderately exacerbated by exertion. Examination of the right upper extremity shows erythema and moderate edema. Capillary refill time is less than 3 seconds. Which of the following is the most likely diagnosis? (A) Axillary-subclavian venous thrombosis (B) Deep venous valvular insufficiency (C) Superficial thrombophlebitis of the basilic vein (D) Superior vena cava syndrome (E) Thoracic outlet syndrome

Answer A is correct. UEDVT most commonly involves thrombosis of the axillary and/or subclavian veins. Symptoms include pain, erythema, and edema of the extremity. Capillary refill is normal. This patient probably has Paget-Schroetter Syndrome. Patients with Paget-Schroetter Syndrome develop spontaneous UEDVT, usually in their dominant arm, after strenuous activity such as rowing, wrestling, weight lifting, or baseball pitching, but are otherwise young and healthy. Answer B is incorrect. Venous valvular insufficiency is a chronic condition that usually occurs in patients with a history of deep vein thrombosis. It is caused by disruption of valvular integrity. Answer C is incorrect. Superficial thrombophlebitis of the basilic vein would present with localized signs of inflammation and not findings of deep vein thrombosis: diffuse pain, erythema, and swelling of an extremity. Answer D is incorrect. Patients with superior vena cava syndrome commonly present with arm and facial edema, head fullness, blurred vision, vertigo, and/or dyspnea. Answer E is incorrect. Patients with thoracic outlet obstruction may have pain that radiates into the fourth and fifth digits via the medial arm and forearm, attributable to injury of the brachial plexus. Symptoms may be position dependent and worsen with hyperabduction of the shoulder or lifting.

A 56-year-old man has had the painful weeping rash shown below for 2 days. He underwent chemotherapy for non-Hodgkin lymphoma 1 year ago. His temperature is 36.7°C (98°F), pulse is 80/min, and blood pressure is 138/76 mm Hg. Examination shows no other abnormalities. Which of the following is the most likely diagnosis? (A) Herpes zoster (B) Impetigo (C) Pyoderma gangrenosum (D) Syphilis (E) Systemic lupus erythematosus

Answer A is correct. Vesicular lesions in a dermatomal pattern suggest zoster. This condition occurs in immunocompetent and immunocompromised individuals. Answer B is incorrect. Impetigo is seen primarily among children, who initially develop a vesicle on the skin surface; this rapidly becomes pustular and breaks down, leaving the characteristic dry, golden crust. Answer C is incorrect. The two classic dermatologic manifestations of inflammatory bowel disease (IBD) are pyoderma gangrenosum and erythema nodosum. Pyoderma gangrenosum appears as a discrete ulcer with a necrotic base, usually on the legs. The ulcer may spread and become large and deep, destroying soft tissues. Pyoderma parallels the activity of the IBD in 50% of cases. Answer D is incorrect. If the primary chancre of primary syphilis is not treated, secondary syphilis may develop 6 to 8 weeks later. This time period can be accelerated in persons infected with HIV. With secondary syphilis, skin, mucous membranes, and lymph nodes are involved. A variety of skin manifestations may occur, including macular, papular, papulosquamous, pustular, follicular, or nodular lesions. Discrete, erythematous, macular lesions over the thorax or hyperpigmented macules on the palms and soles occur most commonly. In moist intertriginous areas, large, pale, flat-topped papules coalesce to form highly infectious plaques or condylomata lata; darkfield microscopy reveals that these lesions are teeming with spirochetes. Mucosal lesions are painless, dull, erythematous patches or grayish-white erosions. These lesions are infectious and darkfield positive as well. Answer E is incorrect. In lupus, oral ulcerations may be painful or painless and are classically located on the tongue or palate. Skin manifestations of lupus are protean and include the classic malar (butterfly) rash, discoid lesions (permanent scarring and disfigurement), alopecia, and photosensitivity.

A 72-year-old woman is brought to the emergency department 2 hours after the sudden onset of visual loss in her right eye. Visual acuity is limited to light perception on the right. Examination of the right eye shows an afferent papillary defect. The anterior structures of both eyes appear normal. Funduscopic findings of the right eye are shown. Which of the following is the most likely diagnosis? (A) Background diabetic retinopathy (B) Central artery occlusion (C) Central vein occlusion (D) Papilledema (E) Proliferative diabetic retinopathy

Answer B is correct. Central retinal artery occlusion Occlusion (usually embolic, but sometimes thrombotic or inflammatory) of the central retinal artery can result in severe vision loss. The main artery supplying the eye is the ophthalmic artery. The part of the ophthalmic artery that enters the eye at the optic nerve is called the central retinal artery. Risk factors for central retinal artery occlusion include atherosclerosis, cardiac disease, and hypercoagulable states. Patients usually complain of the acute painless vision loss. This may have been preceded by transient loss of vision (amaurosis fugax) due to embolization. Occlusion of the central retina artery results in an afferent papillary defect. The affected (ischemic) part of the retina appears white (retinal ganglion cells swell due to ischemia). The foveal center (center of the macula) appears red (called a cherry red spot). This occurs because there are no ganglion cells to swell and obscure the underlying color. Acutely, diagnosis is prompted by the sudden onset of visual acuity loss and the presence of retinal whitening. This image above shows retinal whitening and a "cherry red" spot. Answer A is incorrect. Non-proliferative diabetic retinopathy Non-proliferative diabetic retinopathy, previously called background retinopathy, is the earliest stage of diabetic eye disease. Microscopic changes occur in the blood vessels of the eye in non-proliferative disease; however, the changes typically do not produce symptoms and are not visible to the naked eye. Non-proliferative disease progresses from mild to moderate to severe. Non-proliferative diabetic retinopathy is initially characterized by microaneurysms (microscopic blood-filled bulges in the artery walls) which may burst and leak into the retina. Tiny spots or dots of blood may accumulate in the retina, but they usually do not produce noticeable symptoms in the early stages of the disease. As the disease progresses, hard exudates (accumulations of fluid that has leaked from blood vessels), abnormalities in the growth of microscopic blood vessels in the retina, and bleeding from the veins that feed the retina may occur. While non-proliferative diabetic retinopathy is not itself a sight-threatening condition, it can trigger macular edema or macular ischemia, which can cause rapid vision loss. As the severity of non-proliferative retinopathy increases, the risk of developing sight-threatening proliferative diabetic retinopathy also increases. Answer C is incorrect. Central retinal vein occlusion Central retinal vein occlusion presents with visual loss. The fundus may show retinal hemorrhages, dilated tortuous retinal veins, cotton-wool spots, macular edema, and optic disc edema. Answer D is incorrect. Papilledema Papilledema results from increased intracranial pressure. Signs include venous engorgement, loss of venous pulsation, hemorrhages near the optic disc, blurring of optic margins, elevation of optic disc, and Paton's lines (radial retinal lines cascading from the optic disc.) Answer E is incorrect. Proliferative diabetic retinopathy Proliferative diabetic retinopathy is characterized by neovascularisation - that is, the growth of abnormal new blood vessels in the retina. The vessels are weak and may burst and bleed into the retina or vitreous fluid, causing vision loss.

A 52-year-old man with alcoholism is brought to the emergency department because of a 2-day history of nausea, vomiting, and increasingly severe abdominal pain that radiates to his left shoulder and back. He appears extremely dehydrated and is short of breath. His temperature is 37.8°C (100°F), pulse is 120/min, respirations are 18/min, and blood pressure is 80/60 mm Hg. Abdominal examination shows distention with epigastric tenderness. Bowel sounds are decreased. Rectal examination shows no abnormalities. Test of the stool for occult blood is negative. Laboratory studies show: Hemoglobin 5.5 g/dL Leukocyte count 16,500/mm3 Serum Ca2+ 7.5 mg/dL Amylase 750 U/L A CT scan of the abdomen is shown. Which of the following is the most likely diagnosis? (A) Acute portal vein thrombosis (B) Hemorrhagic pancreatitis (C) Perforated posterior gastric ulcer (D) Ruptured gastric varices (E) Splenic artery rupture

Answer B is correct. Acute pancreatitis should be suspected in a patient with severe abdominal pain that radiates to the back and is associated with an elevated serum amylase level. Alcohol accounts for a substantial number of cases. Patients with acute pancreatitis should be stratified according to prognosis. The patient in this vignette has a poor prognosis as evident by the elevated white blood cell count, the precipitous drop in hemoglobin, and the low serum calcium level. The presence of hypotension and tachycardia are particularly dire. The patient's anemia is severe, suggesting possible hemorrhagic pancreatitis. The diagnosis is confirmed by CT, which shows a heterogeneous mass in the pancreatic bed.

A 27-year-old nulligravid woman comes to the physician because of a 1-year history of irregular heavy menstrual bleeding. She has been otherwise healthy. Menses occur at irregular 15- to 45-day intervals and last 3 to 7 days; menses had previously occurred at regular 28-day intervals with moderate flow. Her last menstrual period was 4 weeks ago. She is sexually active and does not use contraception. Her temperature is 37°C (98.6°F), pulse is 80/min, respirations are 20/min, and blood pressure is 120/80 mm Hg. Physical examination shows no abnormalities. Pelvic examination shows clear cervical mucus. Serum studies show: Thyroid-stimulating hormone 3 μU/mL Follicle-stimulating hormone 5 mIU/mL Prolactin 18 ng/mL Progesterone 0.5 ng/mL (menstrual cycle day 30: follicular N<3; luteal N>3-5) Pelvic ultrasonography shows no abnormalities. Which of the following is the most likely diagnosis? (A) Adenomyosis (B) Anovulation (C) Endometrial polyp (D) Leiomyoma uteri (E) Pregnancy

Answer B is correct. Anovulation is a common cause of irregular bleeding. Hypothalamic anovulation, especially common in young and otherwise healthy women, is characterized by low gonadotropin levels. Typically, the FSH is low or low-normal. (The normal FSH level in a premenopausal woman is 4-30 mIU/mL.) Normal TSH and prolactin levels, as seen in this patient, exclude hypothyroidism and hyperprolactinemia.

A 22-year-old man with a seizure disorder has had increasing cough and shortness of breath for 3 days and fever for 1 day. He has foul-smelling sputum. He had a generalized tonic-clonic seizure 1 week ago. His temperature is 39.4°C (103°F). Crackles are heard on auscultation of the chest. An x-ray of the chest shows a right upper-lobe infiltrate of the lung. Which of the following is the most likely cause? (A) Chemical pneumonitis (B) Pneumonia secondary to anaerobes (C) Pneumonia secondary to gram-negative aerobes (D) Pneumonia secondary to gram-positive aerobes (E) Pneumonia secondary to Mycoplasma pneumoniae

Answer B is correct. Any condition associated with loss of consciousness (e.g., seizure) can predispose a patient to aspiration pneumonia, typically caused by anaerobic bacteria. Foul-smelling sputum in a patient predisposed to aspiration should raise concern for infection with anaerobic bacteria.

A 37-year-old woman comes to the physician for a routine health maintenance examination for insurance purposes. During the past 2 years, she has been tired most of the time, sleeps approximately 9 hours nightly, and occasionally naps on weekends. Three years ago she needed only 7 hours of sleep nightly to feel rested. She has had a 9-kg (20-lb) weight gain during the past 2 years with no change in her diet. She used to swim two to three times weekly; she stopped several months ago because it seemed like too much work to get to the pool. She lives with her mother who has had Parkinson disease for 5 years. The patient has had one serious boyfriend; they dated in college but drifted apart once he moved away after graduation. She has two women friends at work that she sees occasionally. She works as a grade school secretary and has received negative performance assessments during the past year for the number of clerical errors in her work. For 15 years, she has taken an oral contraceptive to regulate her menstrual cycle. She takes no other medications. She does not drink alcohol or use illicit drugs. She is 173 cm (5 ft 8 in) tall and weighs 75 kg (165 lb); BMI is 25 kg/m2. Vital signs are within normal limits. Physical examination shows no abnormalities. On mental status examination, she appears withdrawn but is cooperative. She describes her mood as low. There is no evidence of visual or auditory hallucinations. She says she does not have suicidal ideation but adds, "It doesn't matter. My life isn't worth very much anyway, and nothing is going to change." Results of laboratory studies are within the reference range. Which of the following is the most likely diagnosis? (A) Avoidant personality disorder (B) Dysthymic disorder (C) Primary hypersomnia (D) Schizoid personality disorder (E) Social phobia (F) Substance-induced mood disorder

Answer B is correct. Dysthymic disorder is a condition defined by chronic depressive symptoms of insufficient severity to meet criteria for major depression.

A 60-year-old man has had painful skin with exfoliation of the skin and mucous membranes for 1 day. He has been taking allopurinol and probenecid for 2 weeks because of gouty arthritis. There is diffuse exfoliation of the skin with oozing of serous fluid. The mucous membranes of the mouth are erythematous and exfoliated. There are no target lesions. Which of the following is the most likely diagnosis? (A) Bullous pemphigoid (B) Erythema multiforme (C) Pemphigus vulgaris (D) Staphylococcal scalded-skin syndrome (E) Toxic epidermal necrolysis

Answer E is correct. Painful exfoliation of the skin and mucus membranes two weeks after beginning allopurinol suggests Stevens-Johnson Syndrome or Toxic Epidermal Necrolysis. Answer A is incorrect. Bullous pemphigoid is an acute or chronic autoimmune skin disease, involving the formation of bullae, at the space between the skin layers epidermis and dermis. It is classified as a type II hypersensitivity reaction. Clinically, the earliest lesions may appear urticarial. Tense bullae eventually erupt, most commonly at the inner thighs and upper arms, but the trunk and extremities are frequently both involved. Any part of the skin surface can be involved. Oral lesions are present in a minority of cases. The disease may be acute, but typically will wax and wane. Nikolsky's sign is negative unlike pemphigus vulgaris where it is positive. Very rarely seen in children, bullous pemphigoid most commonly occurs in people 70 years of age and older. In most cases of bullous pemphigoid, no clear precipitating factors are identified. Diagnosis is based on skin biopsy. Glucocorticoids are the mainstay of treatment. Answer B is incorrect. Erythema multiforme is an abrupt, self-limited, but often recurrent eruption of symmetrically distributed papules, plaques, and targetoid erythematous to dusky red lesions that are fixed and have a predilection for the extensor and acral surfaces. Many patients also have oral erosions or targetoid lesions. Vesicles and bullae may evolve from the target lesions. Answer C is incorrect. Pemphigus vulgaris is a chronic blistering skin disease with painful skin lesions. It is an autoimmune disease caused by antibodies directed against both desmoglein 1 and desmoglein 3 resulting in the loss of cohesion between keratinocytes in the epidermis. It is classified as a type II hypersensitivity reaction and characterized by extensive flaccid blisters and mucocutaneous erosions. The severity of the disease, as well as the mucosal lesions, is believed to be directly proportional to the levels of desmoglein 3. Milder forms of pemphigus (like foliacious and erythematoses) are more desmoglein 1 heavy. It arises most often in middle-aged or older people, usually starting with a blister that ruptures easily. The lesions can become quite extensive. The pathogenesis of the disease involves autoantibodies against desmosome proteins, separating keratinocytes from the basal layer of the epidermis. On histology, the basal keratinocytes are usually still attached to the basement membrane. Transudative fluid accumulates in between the keratinocytes and basement membrane (suprabasal split), forming a blister and resulting in what is known as a positive Nikolsky's sign. This is a contrasting feature from bullous pemphigoid, where the detachment occurs between the epidermis and dermis (subepidermal bullae) and where Nikolsky's sign is negative. The gold standard for diagnosis is a punch biopsy from the area around the lesion. Corticosteroids and other immunosuppressive drugs are the mainstay of treatment. Answer D is incorrect. Staphylococcal scalded-skin syndrome (SSSS) is a dermatological condition caused by Staphylococcus aureus. The syndrome is induced by epidermolytic exotoxins (exfoliatin) A and B, which are released by S. aureus and cause detachment within the epidermal layer (by breaking down the desmosomes). These exotoxins are proteases that cleave desmoglein-1, which normally holds the granulosum and spinosum layers together. The disease presents with the widespread formation of fluid filled blisters that are thin walled and easily ruptured and the patient can be positive for Nikolsky's sign. Ritter's Disease of the Newborn is the most severe form of SSSS with similar signs and symptoms. SSSS often includes a widespread painful erythroderma, often involving the face, diaper, and other intertriginous areas. Extensive areas of desquamation might be present. Perioral crusting and fissuring are seen early in the course. Unlike toxic epidermal necrolysis, SSSS spares the mucous membranes. It is most common in children under 6 years, but can be seen in adults who are immunosuppressed or have renal failure. The diagnosis of SSSS is made clinically. This is sometimes confirmed by isolation of S. aureus from blood, mucous membranes, or skin biopsy; however, these are often negative. Skin biopsy may show separation of the superficial layer of the epidermis, differentiating SSSS from TEN, in which the epidermal-dermal layer is separated. The mainstay of treatment is supportive care along with eradication of the primary infection. Conservative measures include rehydration, antipyretics, management of thermal burns, and stabilization. Parenteral antibiotics to cover S. aureus should be administered.

A 21-year-old woman comes to the physician for preconceptional advice. She is recently married and would like to conceive within the next year. She does not eat meat, fish, or dairy products and wishes to decrease the risks of her diet on her baby. Menses occur at regular 28-day intervals and last 5 days. She does not smoke or drink alcohol. She takes no medications. She is 157 cm (5 ft 2 in) tall and weighs 50 kg (110 lb); BMI is 20 kg/m2. Physical examination shows no abnormalities. Pelvic examination shows a normal appearing vagina, cervix, uterus, and adnexa. Which of the following is most likely to decrease the risk of fetal anomalies in this patient? (A) Adjusting diet to include more sources of protein during the first trimester (B) Beginning folic acid supplementation prior to conception (C) Calcium supplementation during the first trimester (D) Iron supplementation during the first trimester (E) Soy protein shakes throughout pregnancy and lactation

Answer B is correct. Folic acid supplementation prior to conception is likely to decrease the risk of neural tube defects.

A 37-year-old man comes to the physician because of nonradiating low back pain for 3 days. The pain began after he worked in his yard. He has not had any change in bowel movements or urination. He had one similar episode 3 years ago that resolved spontaneously. Vital signs are within normal limits. Examination of the back shows bilateral paravertebral muscle spasm. Range of motion is limited by pain. Straight-leg raising is negative. In addition to analgesia, which of the following is the most appropriate next step in management? (A) Bed rest (B) Regular activity (C) X-rays of the spine (D) MRI of the spine (E) Lumbar spine traction

Answer B is correct. In a patient with simple mechanical back pain, regular activity - and no radiographic imaging -- is recommended.

A 5-year-old girl is brought to the physician by her parents for evaluation of recurrent injuries. Her parents say that she started walking at the age of 14 months and since then has always seemed clumsier and had more injuries than other children. She has had increasingly frequent chest pain with exertion since starting a soccer program 3 months ago. She usually has pain or swelling of her knees or ankles after practice. She has been wearing glasses for 2 years. Her 16-year-old brother has required two operations for a severe rotator cuff injury he sustained while taking a shower, and she has a maternal cousin who died of a ruptured aortic aneurysm at the age of 26 years. Today, the patient walks with a limp. She is at the 99th percentile for height and 50th percentile for weight. A midsystolic click is heard at the apex. The left ankle is swollen and tender; range of motion is limited by pain. The joints of the upper and lower extremities are hypermobile, including 25 degrees of genu recurvatum, thumbs that may be extended to touch the forearms, and flexibility at the waist, with palms easily touching the floor with straight knees. Which of the following is the most appropriate next step in diagnosis? (A) Skeletal survey (B) Echocardiography (C) Bone scan (D) MRI of the shoulder (E) Aortic angiography

Answer B is correct. In patients with Marfan syndrome, cardiovascular surveillance includes yearly echocardiograms to monitor the status of the aorta. Surgery to repair the aorta is done when the aortic diameter is greater than 50 mm in adults and older children, when the aortic diameter increases by 10 mm per year, or when there is progressive aortic regurgitation.

A 27-year-old woman, gravida 2, para 1, at 12 weeks' gestation comes to the physician for a prenatal visit. She feels well. Pregnancy and delivery of her first child were uncomplicated. Medications include folic acid and a multivitamin. Her temperature is 37.2°C (99°F), and blood pressure is 108/60 mm Hg. Pelvic examination shows a uterus consistent in size with a 12-week gestation. Urine dipstick shows leukocyte esterase; urinalysis shows WBCs and rare gram-negative rods. Which of the following is the most appropriate next step in management? (A) Recommend drinking 8 oz of cranberry juice daily (B) Oral amoxicillin therapy (C) Oral metronidazole therapy (D) Intravenous cefazolin therapy (E) Intravenous pyelography (F) Cystoscopy

Answer B is correct. In pregnant women, asymptomatic bacteriuria should be treated to prevent ascending infection. In the absence of effective treatment, about one third of pregnant women with asymptomatic bacteriuria will develop acute pyelonephritis. Oral amoxicillin is an appropriate choice. Answer A is incorrect. Cranberry juice is not considered appropriate therapy for asymptomatic bacteriuria in pregnancy. Answer C is incorrect. Metronidazole is often used to treat anaerobic infections. Anaerobic bacteria do not typically cause urinary tract infections. Answer D is incorrect. First-generation cephalosporins, such as cefazolin, are not effective against gram-negative rods. Also, intravenous administration of antibiotic is not needed for asymptomatic bacteriuria, acute cystitis, or mild pyelonephritis. Intravenous administration should be reserved for moderate-to-severe pyelonephritis. Answer E and F are incorrect. Pyelography and cystoscopy are not indicated for asymptomatic bacteriuria.

A 57-year-old woman comes to the physician because of a worsening cough productive of yellow sputum over the past 3 days. She has a 2-year history of shortness of breath with exertion and chronic cough productive of white sputum daily but has no history of pneumonia or other pulmonary problems. She has a 20-year history of hypertension. Current medications include hydrochlorothiazide (25 mg daily) and an ipratropium inhaler (four puffs four times daily). She has smoked one-half pack of cigarettes daily for 37 years. Her temperature is 37°C (98.6°F), pulse is 80/min, respirations are 16/min, and blood pressure is 120/76 mm Hg. Examination shows an increased anteroposterior diameter. Occasional expiratory wheezes are heard. S1 and S2 are normal, and an S4 is heard; the point of maximal impulse is subxiphoid. An x-ray of the chest shows flat diaphragms and a midline cardiac silhouette. Which of the following is the most appropriate next step in management? (A) Perform incentive spirometry (B) Begin oral antibiotic therapy (C) Begin oral theophylline therapy (D) Increase hydrochlorothiazide dosage (E) Switch from hydrochlorothiazide to furosemide therapy

Answer B is correct. Increased anteroposterior diameter, expiratory wheezes, a point of maximal impulse that is subxiphoid, and x-ray showing flat diaphragms and a midline cardiac silhouette - all these findings suggest COPD. The presence of chronic cough productive of white sputum (over years) suggests chronic bronchitis. Exacerbations of chronic bronchitis typically respond to oral antibiotic therapy. Answer A is incorrect. Incentive spiromety is often used post-surgically to improve lung function. Answer C is incorrect. Theophylline is rarely used for COPD nowadays because of its low therapeutic index. Answer D is incorrect. Hydrochlorothiazide is used to treat hypertension and pulmonary edema. High doses confer no additional benefit over low doses and cause additional side effects. Answer E is incorrect. Furosemide relieves pulmonary edema (which this patient does not have).

A 36-year-old nulligravid woman with primary infertility comes for a follow-up examination. She has been unable to conceive for 10 years; analysis of her husband's semen during this period has shown normal sperm counts. Menses occur at regular 28-day intervals and last 5 to 6 days. She is asymptomatic except for severe dysmenorrhea. An endometrial biopsy specimen 5 days before menses shows secretory endometrium. Hysterosalpingography 1 year ago showed normal findings. Pelvic examination shows a normal vagina and cervix. Bimanual examination shows a normal-sized uterus and no palpable adnexal masses. Rectal examination is unremarkable. Which of the following is the most likely diagnosis? (A) Anovulation (B) Endometriosis (C) Intrauterine synechiae (D) Male factor (E) Tubal obstruction

Answer B is correct. Infertility and dysmenorrhea in a nulliparous woman in her 30s is consistent with a diagnosis of endometriosis. Although anovulation is associated with infertility, this diagnosis is excluded by the presence of normal menstrual periods.

A 30-year-old woman comes to the physician because of intermittent throbbing headaches, sweating, and pallor over the past 3 months. She has had several blood pressure measurements that fluctuate from 110/80 mm Hg to 160/108 mm Hg. Her pulse is 100/min, and blood pressure now is 138/88 mm Hg. Serum studies show: Na+ 140 mEq/L Cl− 110 mEq/L K+ 4.5 mEq/L HCO3− 26 mEq/L Urea nitrogen 14 mg/dL Creatinine 1 mg/dL Which of the following is the most likely location of the abnormality? (A) Adrenal cortex (B) Adrenal medulla (C) Aorta (D) Renal arterioles (E) Renal glomeruli (F) Thyroid gland

Answer B is correct. Pheochromocytoma, a tumor of the adrenal medulla, causes hypertension, headache, pallor, and sweating. Blood pressure readings can be quite labile. Answer A is incorrect. Primary aldosteronism causes hypokalemia and is not typically associated with pallor and sweating. Answer C is incorrect. Coarctation of the aorta causes blood pressure to be much higher in the upper extremities than in the lower extremities. It is not typically associated with pallor or sweating. Answer D is incorrect. Renal artery stenosis may produce an abdominal bruit and is not typically associated with pallor or sweating. Answer E is incorrect. Chronic kidney disease is the most common cause of secondary hypertension. It is associated with an elevated serum creatinine level and does not typically cause pallor or sweating. Answer F is incorrect. Hyperthyroidism raises blood pressure and causes sweating, but is not associated with pallor.

A previously healthy 37-year-old woman comes to the physician because of a 2-day history of urinary frequency and urgency and burning with urination. This morning, she noticed some blood in her urine. Her last menstrual period was 3 weeks ago, and she has no abnormal vaginal discharge. She is in a monogamous relationship, and her partner has no similar symptoms. Her temperature is 37.2°C (99°F), pulse is 78/min, respirations are 12/min, and blood pressure is 120/82 mm Hg. Examination shows suprapubic tenderness to palpation without masses, guarding, or rebound. There is no costovertebral angle tenderness. Urine dipstick shows 1+ leukocyte esterase, 2+ blood, and no nitrites. Which of the following is the most likely diagnosis? (A) Acute bacterial urethritis (B) Acute cystitis (C) Atrophic vaginitis (D) Bacterial vaginosis (E) Candidal vaginitis (F) Carcinoma of the bladder (G) Chemical urethritis (H) Chlamydial urethritis (I) Distal ureteral calculus (J) Gonorrheal urethritis (K) Herpes urethritis (L) Interstitial cystitis (M) Pyelonephritis, acute (N) Pyelonephritis, subclinical (O) Trichomonal vaginitis

Answer B is correct. The combination of dysuria (burning with urination), frequency, and urgency in a woman with hematuria, pyuria (1+ leukocyte esterase), and suprapubic tenderness suggests acute bacterial cystitis. Acute bacterial cystitis is the most common cause of dysuria in women.

A 77-year-old man is brought to the physician because of a 12-hour history of word-finding difficulty and weakness and sensory loss of the right arm and leg. He has no history of similar symptoms. He has type 2 diabetes mellitus, hypertension, and atrial fibrillation. Current medications include metformin, lisinopril, and aspirin. He is alert. His pulse is 80/min and irregular, respirations are 16/min, and blood pressure is 170/90 mm Hg. He follows commands but has nonfluent aphasia. There is moderate weakness and decreased sensation of the right upper and lower extremities. Deep tendon reflexes are 2+ bilaterally. Babinski sign is present on the right. His serum glucose concentration is 162 mg/dL. Which of the following is the most appropriate next step in diagnosis? (A) Carotid duplex ultrasonography (B) CT scan of the head (C) EEG (D) Lumbar puncture (E) Cerebral angiography

Answer B is correct. The patient has findings consistent with stroke in the distribution of the left middle cerebral artery. CT scan, the first test recommended in the evaluation of stroke, can distinguish ischemic stroke from hemorrhagic stroke. It can also identify other abnormalities, such as tumor. Answer A is incorrect. Carotid duplex ultrasonography is useful in the work-up of stroke, but should not precede CT evaluation. Answer C is incorrect. Electroencephalography is used to evaluate patients with seizure. Answer D is incorrect. Lumbar puncture is used to evaluate patients with suspected meningitis. Answer E is incorrect. Cerebral angiography is used to evaluate patients with suspected cerebral vasculitis or cerebral aneurysm.

A 62-year-old man comes to the physician because of a 2-month history of progressive fatigue and ankle swelling. He had an anterior myocardial infarction 3 years ago and has had shortness of breath with mild exertion since then. Current medications include labetalol and daily aspirin. He has smoked one-half pack of cigarettes daily for 30 years. His pulse is 100/min and regular, respirations are 20/min, and blood pressure is 130/75 mm Hg. There are jugular venous pulsations 5 cm above the sternal angle. Crackles are heard at both lung bases. Cardiac examination shows an S3 gallop. There is edema from the midtibia to the ankle bilaterally. Further evaluation of this patient is most likely to show which of the following findings? (A) Decreased pulmonary capillary wedge pressure (B) Impaired contractility of the left ventricle (C) Prolapse of the mitral valve (D) Thrombosis of the superior vena cava (E) Ventricular septal defect

Answer B is correct. The patients many signs and symptoms - shortness of breath, fatigue, jugular vein distension, leg edema, an S3, and bibasilar crackles - suggest a diagnosis of heart failure. Prior myocardial infarction would have predisposed the patient to heart failure. Impaired contractility of the left ventricle would be expected.

A 19-year-old college student comes to the physician because of vaginal irritation and pain with urination for 5 days. Two weeks ago, she had streptococcal pharyngitis treated with amoxicillin. She has been sexually active with two partners over the past year; she uses condoms for contraception. Her last menstrual period was 1 week ago. Her temperature is 37.2°C (99°F), and blood pressure is 90/60 mm Hg. Pelvic examination shows erythema of the vulva and vagina and a thick white vaginal discharge. The pH of the discharge is 4. Which of the following is the most likely cause of these findings? (A) Bacterial vaginosis (B) Candidiasis (C) Chlamydia trachomatis infection (D) Escherichia coli infection (E) Neisseria gonorrhoeae infection (F) Trichomoniasis

Answer B is correct. Vulvovaginitis manifested by thick, white vaginal discharge with a pH of 4 suggests candidiasis. Antibiotic administration predisposed this patient to candidiasis. Answer A is incorrect. Bacterial vaginosis causes vulvovaginitis with discharge that has a high pH. Answer C is incorrect. Chlamydia trachomatis causes cervicitis and pelvic inflammatory disease, not vulvovaginitis. Answer D is incorrect. Escherichia coli causes cystitis and pyelonephritis, not vulvovaginitis. Answer E is incorrect. Neisseria gonorrhoea causes cervicitis and pelvic inflammatory disease, not vulvovaginitis. Answer F is incorrect. Trichomonas vaginalis causes vulvovaginitis with discharge that has a high pH.

A 27-year-old woman, gravida 2, para 1, at 10 weeks' gestation comes to the physician for a routine prenatal visit. She is concerned about the risk for Down syndrome in her fetus because her first child has Down syndrome. She would like to be tested as soon as possible. There is no personal or other family history of serious illness. Examination shows a uterus consistent in size with a 10-week gestation. Which of the following is the most appropriate diagnostic test for this syndrome in this patient at this time? (A) Measurement of fetal nuchal fold (B) Measurement of maternal serum α-fetoprotein concentration (C) Chorionic villus sampling (D) Amniocentesis (E) Cordocentesis

Answer C is correct. Chorionic villous sampling is the preferred way to screen for chromosomal abnormalities at 10 to 12 weeks' gestation. Answer A is incorrect. A nuchal scan is an ultrasound used mainly to assess likelihood of Down syndrome. The scan is carried out at 11-13.6 weeks pregnancy and assesses the soft-tissue thickness within the nape of the fetal neck. There are two distinct measurements -- the nuchal translucency, which is measured earlier in pregnancy at the end of the first trimester, and the nuchal fold, which is measured towards the end of the second trimester. Fetuses with Down syndrome tend to have an increased amount of fluid around the neck. Answer B is incorrect. Measurement of maternal serum α-fetoprotein concentration is used to screen for neural tube defects. Answer D is incorrect. Amniocentesis is the preferred way to screen for chromosomal abnormalities at 15 to 16 weeks' gestation. Answer E is incorrect. Percutaneous umbilical cord blood sampling (PUBS), also called cordocentesis, is a diagnostic test that examines blood from the fetal umbilical cord to detect fetal abnormalities. PUBS provides a means of rapid chromosome analysis and is useful when information cannot be obtained through amniocentesis, CVS, or ultrasound (or if the results of these tests were inconclusive). This test carries a significant risk of complication and is typically reserved for pregnancies determined to be at high risk for chromosomal defect. Because the fetal vein is fragile early in pregnancy, PUBS is performed no earlier than 17 weeks into pregnancy. Miscarriage is the primary risk associated with PUBS and occurs in 1-2% of procedures.

A 20-year-old man has had frequent upper respiratory tract infections over the past 4 years. He has daily purulent sputum and has noted decreased exercise tolerance over the past 2 years. He and his wife have been unable to conceive because of his low sperm count. Scattered expiratory wheezing and rhonchi are heard throughout both lung fields. An x-ray of the chest shows hyperinflation. Spirometry shows a decreased FEV1:FVC ratio. Which of the following is most likely to confirm the diagnosis? (A) Arterial blood gas analysis (B) Examination of sputum for eosinophils (C) Sweat chloride test (D) Sputum cytology (E) Bronchoscopy

Answer C is correct. Chronic productive cough, evidence of obstructive lung disease, and azoospermia in a young person suggest a diagnosis of cystic fibrosis. The diagnosis is made by measuring the concentration of chloride in sweat (sweat test).

A 52-year-old man comes to the physician with his wife because of a 1-year history of excessive daytime sleepiness. He does not think the symptoms are problematic, but his wife is concerned because he sometimes falls asleep on the sofa early in the evening when guests are present. He also once fell asleep while driving at night and drove off the road, narrowly avoiding injury. His wife says that he has always snored loudly, and over the past year, he has had episodes of choking or gasping for breath while sleeping. He is 178 cm (5 ft 10 in) tall and weighs 105 kg (231 lb); BMI is 33 kg/m2. His pulse is 76/min, respirations are 14/min, and blood pressure is 150/76 mm Hg. Physical and neurologic examinations show no other abnormalities. Which of the following is most likely to confirm the diagnosis? (A) 24-Hour ambulatory ECG monitoring (B) Multiple sleep latency test (C) Polysomnography (D) CT scan of the head (E) Laryngoscopy

Answer C is correct. Daytime somnolence, loud snoring, and erratic breathing while asleep in an obese, hypertensive man suggests obstructive sleep apnea. Polysomnography should be ordered to confirm the diagnosis and to assess severity.

A 19-year-old college student comes to the physician because of progressive hair growth over her face and body since the age of 16 years. She also has acne and oily skin. Menses have occurred at 30- to 90-day intervals since menarche at the age of 14 years. She has no history of serious illness and takes no medications. She is 168 cm (5 ft 6 in) tall and weighs 88 kg (193 lb); BMI is 31 kg/m2. Her temperature is 37.2°C (99°F), pulse is 72/min, respirations are 16/min, and blood pressure is 120/80 mm Hg. Physical examination shows coarse, pigmented hair over the chin and upper lip, around both nipples, and along the midline of the lower abdomen. The remainder of the examination, including pelvic examination, shows no abnormalities. Serum studies show: Fasting glucose 95 mg/dL (N = 60-100 mg/dL) Fasting insulin 50 μU/mL (N = 6-30 μU/mL) Dehydroepiandrosterone sulfate 3 μg/mL (N = 0.5-5.4) Follicle-stimulating hormone 6 mIU/mL (N = 4-30 mIU/mL) 17α-Hydroxyprogesterone 160 ng/dL (N = 20-300 ng/dL) Luteinizing hormone 35 mIU/mL (N = 5-30 mIU/mL) Testosterone 4.2 nmol/L (N < 3.5 nmol/L) Which of the following is the most appropriate pharmacotherapy? (A) Bromocriptine (B) Clomiphene (C) Combination oral contraceptive (D) Dexamethasone (E) Gonadotropin-releasing hormone agonist

Answer C is correct. Oligomenorrhea and androgen excess (abnormal hair growth and acne) in an overweight adolescent are consistent with a diagnosis of polycystic ovary syndrome. Treatment with oral contraceptives is warranted. Answer A is incorrect. Bromocriptine blocks release of prolactin from the pituitary gland and is used to treat pituitary prolactinomas. Answer B is incorrect. Clomiphene is a selective estrogen receptor modulator (SERM) that increases production of gonadotropins by inhibiting negative feedback on the hypothalamus. It is used to treat female infertility. Answer D is incorrect. Dexamethasone is a high-potency glucocorticoid used to reduce inflammation. Answer E is incorrect. Gonadotropin-releasing hormone agonists are used in a number of conditions to suppress ovarian activity and induce a hypoestrogenic state.

A 57-year-old woman is brought to the emergency department 45 minutes after she fell after an episode of lightheadedness. She has a 6-month history of progressive fatigue, tingling sensations in her fingers and toes, and loss of balance. She underwent a partial gastrectomy for peptic ulcer disease 10 years ago. She has type 2 diabetes mellitus. She has smoked one pack of cigarettes daily for 40 years. Her only medication is insulin. She appears pale. Her temperature is 37°C (98.6°F), pulse is 105/min, respirations are 20/min, and blood pressure is 124/76 mm Hg. The abdomen is soft with a well-healed surgical scar. Sensation to vibration and position is absent over the upper and lower extremities. She has a broad-based gait. Laboratory studies show: Hemoglobin 8.3 g/dL Mean corpuscular volume 105 μm3 Leukocyte count 4800/mm3 Platelet count 100,000/mm3 Serum Bilirubin, total 2.1 mg/dL Direct bilirubin 0.2 mg/dL Lactate dehydrogenase 320 U/L Which of the following is the most likely explanation for these findings? (A) Amyotrophic lateral sclerosis (B) Diabetes mellitus (C) History of gastrectomy (D) Meniere disease (E) Multiple sclerosis (F) Vertebrobasilar deficiency

Answer C is correct. Paresthesias, ataxia, and large-fiber sensory loss (manifested by absent vibration and proprioception in the feet), combined with a macrocytic anemia and hemolysis (manifested by elevated unconjugated bilirubin and LDH levels) point to vitamin B12 deficiency. This was most likely caused by lack of intrinsic factor due to prior gastrectomy.

A previously healthy 42-year-old woman comes to the physician because of a 6-month history of difficulty swallowing and regurgitation of undigested food. She also has had color changes and pain in her fingertips when they are exposed to cold temperatures. She has not had nausea, abdominal pain, diarrhea, or constipation. Examination of the face shows thick, tightly bound skin. The fingertips are tapered and covered with ischemic ulcerations. There are telangiectasias diffusely distributed over the trunk and upper extremities. Funduscopic examination shows retinal hemorrhages bilaterally. Which of the following is the most appropriate pharmacotherapy for this patient's pain and ulcerations? (A) Enalapril (B) Methotrexate (C) Nifedipine (D) Prednisone (E) Propranolol

Answer C is correct. Skin thickening and tightening, along with telangiectasias, Raynaud phenomenon, dysphagia, and esophageal reflux, suggests scleroderma. Nearly all patients with scleroderma experience Raynaud phenomenon, which has caused digital ulcerations in this patients. Long-acting nifedipine is an effective therapy for this condition. If this patient were to develop scleroderma renal crisis, an ACE-inhibitor (e.g., enalapril) would be indicated.

A 22-year-old man comes to the physician for a routine health maintenance examination. He feels well. He has had a painless left scrotal mass since childhood. Examination shows a 6-cm, soft, nontender left scrotal mass that transilluminates; there are no bowel sounds in the mass. Examination of the testis shows no abnormalities. Which of the following is the most likely cause of the mass? (A) Accumulation of scrotal adipose tissue (B) Cryptorchidism of the left testis (C) Dilation of the pampiniform plexus of veins around the testis (D) Persistence of a patent processus vaginalis (E) Torsion of the left testis

Answer D is correct. A hydrocele is a collection of fluid in either the tunica vaginalis or a persistent processus vaginalis. Hydroceles are common in newborns, but may appear at any age as a painless, unilateral scrotal swelling of acute or insidious onset. On examination, a hydrocele is fluctuant, ovoid, and nontender. Transillumination (shining a light through the swelling) demonstrates light transmission through the fluid-filled hydrocele, compared with nontransmission or limited transmission through the solid testis. Answer A is incorrect. Adipose tissue does not transilluminate. Answer B is incorrect. Cryptorchidism does not present as a scrotal mass. Answer C is incorrect. A varicocele is a dilation of the pampiniform venous plexus along the spermatic cord. It usually occurs on the left side. Patients may be asymptomatic or may complain of a dull, dragging discomfort on the affected side. Palpation reveals the classic "bag of worms" along the spermatic cord. The mass disappears when the patient lies down and reappears when the patient stands. Answer E is incorrect. Patients with testicular torsion typically present with sudden onset of painful testicular swelling. The pain radiates to the inguinal and hypogastric areas and may be accompanied by nausea and vomiting. The testis will be tender and swollen, and is often high in the scrotum because of shortening of the spermatic cord with torsion.

A 46-year-old man comes to the physician because of intermittent lower abdominal pain over the past 3 months. There is no family history of cancer. Examination shows no other abnormalities. His hematocrit is 38%. Test of the stool for occult blood is positive. Colon contrast studies show a 1.5-cm polyp in the descending colon. An upper gastrointestinal series shows no abnormalities. Which of the following is the most appropriate next step in management? (A) CT scan of abdomen (B) Repeat test of the stool for occult blood after 3 days on a meat-free diet (C) Measurement of serum carcinoembryonic antigen (CEA) concentration (D) Colonoscopy with polypectomy (E) Total colectomy

Answer D is correct. All patients with symptoms suggestive of colorectal neoplasia should undergo an evaluation of the colon by colonoscopy, flexible sigmoidoscopy, or double contrast barium enema. Colonoscopy has greater accuracy than a barium enema study in the detection of small polyps and early cancers as well as the ability to remove neoplasms or biopsy lesions at the time of the examination. Lesions detected on barium enema study necessitate colonoscopic evaluation. In patients with actual colorectal cancer, CT scanning and of the abdomen and pelvis is used preoperatively to assess the extent of metastatic disease. Magnetic resonance scanning and positron emission tomography may also be useful in detecting metastatic disease in select patients. EUS is used for the preoperative staging of rectal cancer. Carcinoembryonic antigen level is measured preoperatively for a baseline value and, if elevated, monitored to detect tumor recurrence postoperatively.

A 10-year-old boy is brought for a follow-up examination 2 days after he was seen in the emergency department because of hives, hoarseness, and light-headedness. His symptoms began 15 minutes after he was stung by a bee and lasted approximately 60 minutes; they resolved before he was treated. He has been stung by bees three times over the past year, and each reaction has been more severe. Examination shows no abnormalities. Which of the following is the most appropriate recommendation to prevent future morbidity and mortality from this condition? (A) Avoid areas known to have bees (B) Avoid wearing colorful clothing outside (C) Carrying diphenhydramine tablets (D) Carrying self-injectable epinephrine (E) Seek immediate medical attention following any future sting

Answer D is correct. Bee stings can cause anaphylaxis. Any child with a previous severe anaphylactic reaction should be given an epinephrine autoinjector (EpiPen, Twinject) and a written emergency plan in case of re-exposure.

A 3-year-old girl is brought to the physician because of fever and left ear pain for 3 days. She has been treated with amoxicillin for the past 5 days for left otitis media. Her temperature is 38.5°C (101.3°F), pulse is 100/min, respirations are 20/min, and blood pressure is 80/60 mm Hg. Examination shows the left ear displaced forward and laterally from the head. There is edema and tenderness behind the left ear. Otoscopic examination shows a red, dull, left tympanic membrane that does not move. Which of the following is the most likely diagnosis? (A) Acoustic neuroma (B) Labyrinthitis (C) Lateral sinus thrombosis (D) Mastoiditis (E) Rhabdomyosarcoma

Answer D is correct. Fever and ear pain associated with otitis media and displacement (forward and lateral) of the ear suggest a diagnosis of mastoiditis. Answer A is incorrect. An acoustic neuroma (or more precisely, vestibular schwannoma) is a benign tumor that arises from the Schwann cells of cranial nerve VIII. Acoustic neuromas account for about 10% of all intracranial tumors. They are most commonly diagnosed in middle age. They are slightly more common in women than men. They are usually sporadic but may be associated with neurofibromatosis 1 or 2 (NF-1, NF-2). Most patients with NF-2 will develop bilateral acoustic neuromas. Acoustic neuromas in NF-1 are much less common. The primary symptoms of vestibular schwannoma are asymmetric hearing loss (sensorineural) and tinnitus. The hearing loss is usually gradual in onset and progressive but can occur suddenly. Disequilibrium is not usually the chief complaint on presentation, but patients often admit to mild unsteadiness. Larger tumors can cause dysesthesia around the ear or facial weakness, or both. If the neuroma is diagnosed late, patients can manifest cerebellar symptoms and symptoms of mass effect and obstructing hydrocephalus. After a complete neuro-otologic examination and audiologic evaluation, an MRI scan of the brain with fine cuts through the internal auditory canal with gadolinium contrast is necessary for diagnosis. Treatment options include observation, surgery, or stereotactic radiotherapy. Most vestibular schwannomas require treatment to prevent cerebral complications from future growth. Answer B is incorrect. Labyrinthitis causes sudden and severe vertigo, tinnitus, and hearing loss. The hearing loss is sensorineural, is often severe, and can be permanent. Labyrinthitis is caused by inflammation within the inner ear. The cause is most often a viral infection but can be bacterial. Bacterial labyrinthitis usually results from extension of a bacterial otitis media into the inner ear. A noninfectious serous labyrinthitis can also occur after an episode of acute otitis media. Other, less common causes include treponemal infections (syphilis) and rickettsial infection (Lyme disease). Antibiotics are recommended if a bacterial cause is suspected. As with acute otitis media, bacterial labyrinthitis can, in rare cases, lead to meningitis. Few other conditions cause the constellation of hearing loss, tinnitus, and vertigo, but cerebrovascular ischemia, meningitis, brain abscess, and encephalitis should all be considered. Although the vertigo should resolve over days to weeks, hearing loss and tinnitus can persist. Drugs known to be ototoxic can cause acute onset of hearing loss and disequilibrium, although this is not true labyrinthitis. These drugs include salicylates, aminoglycosides, loop diuretics, and various chemotherapeutic agents. This cause should be considered in patients who complain of hearing loss or dizziness while taking these medications. Answer C is incorrect. Septic thrombosis of the lateral sinus results from acute or chronic infections of the middle ear. Symptoms consist of ear pain and fever followed by headache, nausea, vomiting, loss of hearing, and vertigo, usually evolving over a period of several weeks. Symptoms or signs suggestive of otitis media, including mastoid swelling, may be seen. Sixth cranial nerve palsies can occur, but other focal neurologic signs are rare. In some patients with nonseptic lateral sinus thrombosis, headache may be the only symptom. Papilledema occurs in 50% of cases, and elevated CSF pressure is present in most. Answer E is incorrect. In rhabdomyosarcoma, cancer cells arise from skeletal muscle progenitors. Most tumors of this type occur in the head, neck, or genitourinary tract. Rhabdomyosarcoma is a relatively rare form of cancer. It is most common in children ages one to five.

A 27-year-old woman, gravida 3, para 2, at 25 weeks' gestation is admitted to the hospital 30 minutes after spontaneous rupture of membranes. She reports that after walking a 5-km course 24 hours ago, she began to have muscle aches and chills. Her temperature prior to going to bed last night was 38.9°C (102°F). She took 800 mg of ibuprofen and awoke 8 hours later with severe abdominal pain and shaking chills. She began to vomit, and then noted a gush of blood-tinged fluid from her vagina. Her pregnancy had been uncomplicated. On admission, she appears ill. Her temperature is 39.1°C (102.3°F), pulse is 110/min, respirations are 18/min, and blood pressure is 110/60 mm Hg. Abdominal examination shows severe, diffuse tenderness throughout the lower quadrants. Pelvic examination shows exquisite cervical motion and uterine tenderness. Speculum examination confirms rupture of membranes. Ultrasonography shows the fetus in a vertex presentation. There is anhydramnios. Which of the following is the greatest predisposing factor for this patient's premature rupture of membranes? (A) Emesis (B) Exercise (C) Ibuprofen use (D) Intra-amniotic infection (E) Multiparity

Answer D is correct. Fever, chills, abdominal pain, and cervical motion and uterine tenderness in a pregnant woman with ruptured membranes suggest chorioamnionitis.

A 27-year-old woman comes to the physician because she wants to begin a sex change process. She explains that she has always felt "more like a boy than a girl." Since childhood, she has dressed in masculine clothes and has preferred to participate in traditionally male-dominated activities. She says that she is sexually attracted to men despite her masculine tendencies. Physical examination shows no abnormalities. On mental status examination, she is calm with a euthymic mood and full range of affect. Her thought process is organized. Which of the following is the most likely diagnosis? (A) Body dysmorphic disorder (B) Delusional disorder (C) Female orgasmic disorder (D) Gender identity disorder (E) Transvestic fetishism

Answer D is correct. Gender identity disorder is a condition wherein a person experiences a strong, persistent desire to be of the opposite sex or insists that he or she is in actuality of the opposite sex.

A 32-year-old woman comes to the physician because of fatigue for 6 months. She has had progressively severe dyspnea on exertion for 6 weeks. She had an extensive abdominal operation 5 years ago for Crohn disease. She does not take any medications. Her temperature is 37°C (98.6°F), pulse is 62/min, respirations are 18/min, and blood pressure is 110/65 mm Hg. Examination of the thyroid gland, lungs, heart, abdomen, and extremities shows no abnormalities. Test of the stool for occult blood is negative. Laboratory studies show: Hemoglobin 8 g/dL Mean corpuscular volume 70 μm3 Leukocyte count 9000/mm3 Platelet count 500,000/mm3 Which of the following is the most likely diagnosis? (A) Acute leukemia (B) Anemia of chronic disease (C) Folic acid deficiency (D) Iron deficiency (E) Lyme disease (F) Microangiopathic hemolytic anemia (G) Pernicious anemia (H) Sleep apnea

Answer D is correct. Iron deficiency anemia (due to blood loss or malabsorption) is the most common cause of microcytic anemia. A reactive thrombocytosis occurs typically. Answer A is incorrect. Acute leukemia is associated with a normocytic anemia. The leukocyte count is usually abnormal. Answer B is incorrect. The anemia of chronic disease is usually normocytic. It can be microcytic, but MCVs as low as 70 are rare. Moreover, the anemia of chronic disease produces a mild-to-moderate anemia. Severe anemia with a Hgb of 8 gm/dL would be highly unusual. Answer C is incorrect. Folic acid deficiency is associated with macrocytic anemia. Answer E is incorrect. Lyme disease, if chronic, can cause anemia of chronic disease. Anemia of chronic disease is typically associated with chronic infection or inflammation, connective tissue disease, or neoplasia. Answer F is incorrect. Microangiopathic hemolytic anemia is associated with normocytic or macrocytic anemia. Schistocytes are present on peripheral blood smear. Answer G is incorrect. Pernicious anemia, due to vitamin B12 deficiency, is associated with macrocytic anemia. Answer H is incorrect. Sleep apnea is associated with polycythemia, not anemia.

A 52-year-old woman has had dyspnea and hemoptysis for 1 month. She has a history of rheumatic fever as a child and has had a cardiac murmur since early adulthood. Her temperature is 36.7°C (98°F), pulse is 130/min and irregularly irregular, respirations are 20/min, and blood pressure is 98/60 mm Hg. Jugular venous pressure is not increased. Bilateral crackles are heard at the lung bases. There is an opening snap followed by a low-pitched diastolic murmur at the third left intercostal space. An x-ray of the chest shows left atrial enlargement, a straight left cardiac border, and pulmonary venous engorgement. Which of the following is the most likely explanation for these findings? (A) Aortic valve insufficiency (B) Aortic valve stenosis (C) Mitral valve insufficiency (D) Mitral valve stenosis (E) Tricuspid valve insufficiency

Answer D is correct. Mitral stenosis is associated with an opening snap followed by a low-pitched diastolic murmur at the third left intercostal space. It can result in left atrial enlargement, pulmonary venous engorgement, and hemoptysis (ruptured capillaries due to in increased hydrostatic pressure). Left atrial enlargement predisposes to atrial fibrillation, which is associated with an irregularly irregular heart beat and worsened pulmonary edema. Answer A is incorrect. Aortic insufficiency is associated with a high-pitched, blowing, early diastolic, decrescendo murmur at 2nd to 4th left interspace with radiation to apex Answer B is incorrect. Aortic stenosis is associated with a loud, harsh midsystolic murmur at the right 2nd interspace with radiation to neck and down left sternal border toward apex. Answer C is incorrect. Mitral insufficency is associated with a blowing, holosystolic murmur at apex radiating to left axilla. Answer E is incorrect. Tricuspid insufficiency is associated with a blowing, holosystolic murmur at the lower left sternal border that increases with inspiration

A 32-year-old woman, gravida 2, para 1, at 8 weeks' gestation comes to the physician for her first prenatal visit. She delivered her first child spontaneously at 34 weeks' gestation; pregnancy was complicated by iron deficiency anemia. She has no other history of serious illness. Her blood pressure is 100/70 mm Hg. Examination shows no abnormalities. Ultrasonography shows a dichorionic-diamniotic twin intrauterine pregnancy consistent in size with an 8-week gestation. This patient is at increased risk for which of the following complications? (A) Abruptio placentae (B) Fetal chromosome abnormality (C) Hyperthyroidism (D) Preterm labor and delivery (E) Twin transfusion syndrome

Answer D is correct. Multiple gestation carries an increased risk of preterm labor and delivery.

A 27-year-old man comes to the physician because of persistent headaches since a motor vehicle collision 2 weeks ago. In the collision, he struck his head on the windshield but did not lose consciousness. The headaches are generalized with a constant pressure-like pain. He also has had memory loss and difficulty sleeping. Neurologic examination and an MRI of the brain show no abnormalities. Which of the following is the most likely diagnosis? (A) Increased intracranial pressure (B) Migraine (C) Occipital neuralgia (D) Post-traumatic headache (E) Subarachnoid hemorrhage (F) Subdural hematoma (G) Temporomandibular joint syndrome (H) Tension-type headache

Answer D is correct. Post-traumatic headache is associated with irritability, concentration impairment, insomnia, memory disturbance, and light-headedness. Anxiety and depression are present to variable degrees. Neurological examination and MRI of the brain would be normal.

A 6-year-old girl is brought to the physician because of a 1-month history of a recurrent pruritic rash on her arms. She was born at term and has been healthy except for an episode of wheezing 6 months ago treated with albuterol. A photograph of the rash is shown. Which of the following is the most appropriate next step in management? (A) Coal tar therapy (B) Oral antibiotic therapy (C) Topical antibiotic therapy (D) Topical corticosteroid therapy (E) Vitamin supplementation

Answer D is correct. Recurrent pruritic rash on the flexor surfaces of the elbows in a child with possible atopy suggests atopic dermatitis. Topical corticosteroids are the mainstay of therapy.

A previously healthy 17-year-old girl comes to the physician because of a 1-week history of itching and progressive rash. She has no history of skin problems or associated symptoms. She takes no medications. Her sister with whom she shares a room had similar symptoms during the previous week. The patient's temperature is 36.8°C (98.2°F). There are multiple 2- to 5-mm erythematous papules over the trunk, especially at the waistline, and over the forearms, hands, and fingers. There is no lymphadenopathy or hepatosplenomegaly. Which of the following is the most likely causal organism? (A) Epstein-Barr virus (B) Group A streptococcus (C) Measles virus (D) Sarcoptes scabiei (E) Varicella-zoster virus

Answer D is correct. Scabies is a common cause of rash and itching, especially in young people. Communicability between roommates is typical and involvement of the fingers, hands, forearms, and waistline is often observed. Scabies produces no signs of systemic illness, unlike infection with Epstein-Barr virus, Group A streptococcus, measles virus, and varicella-zoster virus.

A 17-year-old girl comes to the physician 5 days after she was found to have a hemoglobin concentration of 9 g/dL during a school blood drive. She has a history of frequent nosebleeds. Menarche began at the age of 15 years, and her menses have occurred at regular 28-day intervals and last 9 to 10 days; she has heavy bleeding on all but the last day. Her mother and older sister have similar symptoms. Physical and pelvic examinations show no abnormalities. Today, her hemoglobin concentration is 9 g/dL, leukocyte count is 8000/mm3, and platelet count is 300,000/mm3. Which of the following conditions is most likely in this patient? A. Factor VIII deficiency B. Factor IX deficiency C. Fibrinogen deficiency D. Protein C deficiency E. Protein S deficiency F. Thrombocytopenia G. Vitamin K deficiency H. von Willebrand factor deficiency

Answer H is correct. Von Willebrand disease is the most common hereditary coagulation abnormality described in humans. It affects females and males. There are three types of hereditary vWD: vWD Type I, vWD Type II, and vWD Type III. vWD Type I is the most common type those that have it are typically asymptomatic or may experience mild symptoms such as nosebleeds. The various types of vWD present with varying degrees of bleeding tendency, usually in the form of easy bruising, nosebleeds and bleeding gums. Women may experience heavy menstrual periods and blood loss during childbirth. Severe internal or joint bleeding is uncommon. Answer A is incorrect. Factor VIII deficiency (hemophilia A) is an X-linked disorder and, therefore, occurs almost exclusively in males. Symptoms and signs of severe deficiency develop in childhood with bleeding into muscles, joints, and soft tissues. Bleeding is often spontaneous and is common after any type of surgery or even mild injury. Bleeding often occurs in joints and in the retroperitoneum. Hematuria and mucosal and intracranial bleeding also occur. Patients with moderate hemophilia have less spontaneous bleeding, but are at significant risk of bleeding after surgery or trauma. Patients with mild hemophilia may be undetected in adulthood and diagnosed only with bleeding after major surgery. Answer B is incorrect. Factor IX deficiency (hemophilia B) is an X-linked disorder and, therefore, occurs almost exclusively in males. Symptoms and signs of severe deficiency develop in childhood with bleeding into muscles, joints, and soft tissues. Bleeding is often spontaneous and is common after any type of surgery or even mild injury. Bleeding often occurs in joints and in the retroperitoneum. Hematuria and mucosal and intracranial bleeding also occur. Patients with moderate hemophilia have less spontaneous bleeding, but are at significant risk of bleeding after surgery or trauma. Patients with mild hemophilia may be undetected in adulthood and diagnosed only with bleeding after major surgery. Answer C is incorrect. Factor I (or fibrinogen) deficiency is a very rare inherited disorder with complications that vary with the severity of the disorder. Answer D is incorrect. Protein C deficiency predisposes to clotting, not to bleeding. Inherited risk factors for venous thrombosis include Factor V Leiden, Prothrombin G20210A, and deficiencies of naturally occurring anticoagulants (antithrombin III, protein C, and protein S). Protein C deficiency is less common than Factor V Leiden and Prothrombin G20210A, but is more likely to produce symptomatic venous thrombosis at an early age. Answer E is incorrect. Protein S deficiency predisposes to clotting, not to bleeding. Inherited risk factors for venous thrombosis include Factor V Leiden, Prothrombin G20210A, and deficiencies of naturally occurring anticoagulants (antithrombin III, protein C, and protein S). Protein S deficiency is less common than Factor V Leiden and Prothrombin G20210A, but is more likely to produce symptomatic venous thrombosis at an early age. Answer F is incorrect. The patient in this vignette has a normal platelet count. Answer G is incorrect. Inpatients and outpatients who are severely ill may have bleeding resulting from acquired coagulation factor deficiencies. Foremost among the causes of low factor levels is vitamin K deficiency. Vitamin K deficiency may be caused by (1) biliary tract disease interfering with enterohepatic circulation, leading to decreased absorption of vitamin K; (2) drugs, especially antibiotics, that sterilize the gut and reduce bacterial sources of vitamin K or other drugs (cholestyramine) that directly block vitamin K absorption; and (3) poor nutritional status induced by malabsorption, chronic disease, or poor oral intake in patients who are acutely ill. As noted previously, factors II, VII, IX, and X are vitamin K-dependent procoagulant factors, as are proteins C and S. Warfarin blocks vitamin K-dependent γ-carboxylation of these factors and causes an acute decrease in functional factor VII levels because factor VII has the shortest half-life (6 hours) of all vitamin K-dependent factors in vivo. The patient in this vignette is not predisposed to vitamin K deficiency.

A 72-year-old woman with unresectable small cell carcinoma of the lung is brought to the emergency department after her family found her unresponsive. During the past week, she has become progressively confused. On arrival, she does not respond to command but withdraws all extremities to pain. Her temperature is 37°C (98.6°F), pulse is 80/min, respirations are 12/min, and blood pressure is 130/70 mm Hg. The pupils are equal and reactive to light, and corneal reflexes are brisk; there is spontaneous medial and lateral gaze. Laboratory studies show: Hemoglobin 12.2 g/dL Leukocyte count 6000/mm3 Na+ 118 mEq/L Cl− 98 mEq/L K+ 4.5 mEq/L HCO3− 26 mEq/L Urea nitrogen 16 mg/dL Glucose 95 mg/dL Creatinine 0.8 mg/dL Which of the following is the most likely mechanism of these findings? A. Adrenal insufficiency B. Inadequate renal blood flow C. Injury to the renal tubules D. Nonphysiologic ADH (vasopressin) secretion E. Nonphysiologic aldosterone secretion F. Physiologic ADH (vasopressin) secretion G. Physiologic aldosterone secretion

Answer D is correct. Small cell carcinoma of the lung is associated with SIADH, which results in hyponatremia. SIADH is characterized by nonphysiologic ADH (vasopressin) secretion.

A 47-year-old man comes to the physician 12 hours after the sudden onset of a severe occipital headache and stiff neck. He has not had any other symptoms and has no history of severe headache. He has hypertension and gastroesophageal reflux disease. Current medications include hydrochlorothiazide and ranitidine. He is oriented to person, place, and time. His temperature is 36.7°C (98.1°F), pulse is 100/min, and blood pressure is 160/90 mm Hg. Range of motion of the neck is decreased due to pain. Neurologic examination shows no focal findings. Which of the following is the most likely diagnosis? (A) Cluster headache (B) Meningitis (C) Migraine (D) Subarachnoid hemorrhage (E) Tension-type headache

Answer D is correct. Sudden onset of a severe occipital headache and neck stiffness, particularly in an individual with no history of severe headache, is worrisome for subarachnoid hemorrhage. The absence of focal findings on neurological exam is not surprising. This patient should undergo CT scanning of brain. If the CT scan is normal, lumbar puncture should be undertaken and the cerebrospinal fluid should be centrifuged to detect xanthochromia. Answer A is incorrect. The pain in cluster headache is of extreme intensity, is unilateral, and is associated with congestion of the nasal mucosa and injection of the conjunctiva on the side of the pain. Increased sweating of the ipsilateral side of the forehead and face may occur. There may be associated ocular signs of Horner syndrome: miosis, ptosis, and the additional feature of eyelid edema. The pain is usually steady, nonthrobbing, and invariably localized retro-orbitally on one side of the head; it may occasionally spread to the ipsilateral side of the face or neck. Attacks often awaken patients, usually 2 to 3 hours after the onset of sleep ("alarm-clock headache"). The duration of headache is usually around 1 hour, although it may recur several times in a day, recurring paroxysmally (in clusters) for several weeks. Answer B is incorrect. Patients with bacterial meningitis may exhibit fever, headache, lethargy, confusion, irritability, and stiff neck. One of three principal modes of onset can occur. (1) Approximately 25% of cases begin abruptly with fulminant illness; mortality is very high in this setting. (2) More often, meningeal symptoms progress over 1 to 7 days. (3) Meningitis may superimpose itself on 1 to 3 weeks of an upper respiratory-type illness; diagnosis is most difficult in this group. Occasionally, no more than a single additional neurologic symptom or sign hints at disease more serious than a routine upper respiratory tract infection. Stiff neck is absent in roughly one half of all patients with meningitis, notably in the very young, the old, and the comatose. A petechial or purpuric rash is found in one half of patients with meningococcemia; although not pathognomonic, palpable purpura is very suggestive of N. meningitidis infection. Approximately 20% of patients with acute bacterial meningitis have seizures, and a similar fraction has focal neurologic findings. The absence of fever in this patient goes against a diagnosis of meningitis. Answer C is incorrect. The two most common types of migraine are migraine without aura and migraine with aura; migraine without aura accounts for 80% of patients. Migraine auras are focal neurologic symptoms that precede, accompany, or, rarely, follow an attack. The aura usually develops over 5 to 20 minutes, lasts less than 60 minutes, and can involve visual, sensorimotor, language or brainstem disturbances. The most common aura is typified by positive visual phenomena (such as scintillating scotomata) that often precede the headache; they resemble the effect of being too close to a photographer with a flash camera (phosphenes). The pain of migraine is often pulsating, unilateral, and frontotemporal in distribution and often accompanied by anorexia, nausea, and, occasionally, vomiting. In characteristic attacks, patients are markedly intolerant of light (photophobia) and seek rest in a dark room. There may also be intolerance to sound (phonophobia) and occasionally to odors (osmophobia). The diagnosis of migraine requires the presence of at least one of these features, particularly in the absence of gastrointestinal symptoms. The presence of these symptoms results in a syndrome that is invariably disabling for the patient, to the extent that for the duration of the attack he or she is unable to function normally. In children, migraine is often associated with episodic abdominal pain, motion sickness, and sleep disturbances. Onset of typical migraine late in life (older than age 50) is rare, although recurrence of migraine that had been in remission is not uncommon. Recurrent migraine headache associated with transient hemiparesis or hemiplegia occurs rarely as a clearly genetically determined (Mendelian) disease (familial hemiplegic migraine). In this patient, the location of the headache, presence of neck stiffness, and absence of associated symptoms argue against the diagnosis of migraine. Also, it would be unusual for a migraine headache to appear for the first time in a 47-year-old man. Answer E is incorrect. In contrast to migraine, pain of tension-type headache is usually not throbbing but rather steady and often described as a "pressure feeling" or a "viselike" sensation. It is usually not unilateral and may be frontal, occipital, or generalized. There is frequently pain in the neck area, unlike in migraine. Pain commonly lasts for long periods of time (e.g., days) and does not rapidly appear and disappear in attacks. There is no "aura." Photophobia and phonophobia are usually absent. Although tension-type headache may be related by the patient to occur or be exacerbated at times of particular emotional stress, the pathophysiology may relate to sustained craniocervical muscle contraction; hence, a more appropriate term for this syndrome is muscle-contraction headache. A careful evaluation should be made of the patient's psychosocial milieu and the presence of anxiety or depression. The tricyclic antidepressant drugs in low doses have proven the most useful for prevention of tension-type headache; although the best documented is amitriptyline, newer agents with fewer side effects may be equally effective. Nonpharmacologic therapies such as relaxation therapy, massage, physiotherapy, or acupuncture may be useful in refractory cases. The presence of sudden-onset, severe headache with neck stiffness in this patient is not suggestive of tension-type headache.

A 4-year-old boy with asthma becomes limp during treatment with inhaled albuterol in the emergency department. Ten minutes ago, he received intravenous methylprednisolone for an acute exacerbation, and he was alert and oriented at that time. He received the diagnosis of asthma 2 years ago and has been admitted to the hospital for acute exacerbations eight times since then. Current medications include albuterol and montelukast. He appears pale and gasps and moans as he attempts to breathe. He responds to voice. His temperature is 36°C (96.8°F), pulse is 160/min and thready, respirations are 18/min, and blood pressure is 50/20 mm Hg. The skin is cold to the touch. Pulmonary examination shows poor air movement bilaterally. No wheezes are heard. The point of maximal impulse is 2 cm to the left of the midclavicular line in the sixth intercostal space. A chest x-ray is shown. Which of the following is the most likely underlying cause of this patient's hypotension? (A) Adverse effect of albuterol (B) Adverse effect of methylprednisolone (C) Atelectasis of the left lung (D) Decrease in cardiac output (E) Severe bronchospasm

Answer D is correct. The chest x-ray reveals hyperlucency and absence of vascular markings in the right thoracic space, shift of the heart to the left, and left tracheal deviation. These indicate tension pneumothorax, which causes hypotension by decreasing cardiac output. Asthma may have predisposed the child to pneumothorax.

A previously healthy 27-year-old nulligravid woman comes to the emergency department because of a 2-day history of moderate-to-severe pain and swelling of the left labia. She is sexually active and uses condoms inconsistently. Her temperature is 37.2°C (99°F), pulse is 92/min, respirations are 18/min, and blood pressure is 115/75 mm Hg. Pelvic examination shows a 4 x 3-cm, tender, fluctuant mass medial to the left labium majus compromising the introital opening. Which of the following is the most appropriate next step in management? (A) Administration of intravenous metronidazole (B) Administration of intravenous penicillin G (C) Ultrasound-guided needle aspiration of the mass (D) Incision and drainage (E) Vulvectomy

Answer D is correct. The main physical finding - a tender, fluctuant mass medial to the left labium majus compromising the introital opening - is indicative of a Bartholin cyst. The patient in the vignette is symptomatic, making incision and drainage the best option for treatment.

A previously healthy 3-week-old newborn is brought to the emergency department because of a 2-day history of irritability and yellow skin. During this period, he has had dark urine and pale stools. He was born at term following an uncomplicated pregnancy and delivery. His mother was immunized against hepatitis B prior to conception. He is at the 50th percentile for length, weight, and head circumference. His temperature is 37.3°C (99.1°F), pulse is 120/min, respirations are 40/min, and blood pressure is 70/30 mm Hg. His skin is diffusely pale yellow. Abdominal examination shows mild distention and a subhepatic mass. What is the most likely mechanism of jaundice? (A) Autoimmune disorder (B) Bacterial infection (C) Metabolic process (D) Obstruction (E) Physiologic process (F) Toxin exposure

Answer D is correct. The newborn in the case vignette is three weeks old. This is already outside the boundary for physiologic and breastfeeding jaundice (which occur in the first week of life.) Infants with breast milk jaundice are well appearing and not irritable. The fact that she is irritable and has abdominal distension and a subhepatic mass only adds to this conclusion. There are many pathologic causes of jaundice in the newborn. The presence of a subhepatic mass in this case suggests obstruction of the biliary tract as a likely cause. (No other options are associated with subhepatic mass.) Answer A is incorrect. Autoimmune hepatitis, primary biliary cirrhosis, and sclerosing cholangitis are examples of autoimmune diseases that cause jaundice. These conditions, however, do not affect neonates and do not produce abdominal mass. Answer B is incorrect. Sepsis can cause jaundice in newborns. The clinical presentation can be subtle, which is why newborns with fever are usually treated empirically for sepsis while work-up is underway. The newborn in the case vignette is afebrile and bacterial infection would not be associated with an abdominal mass. Answer C is incorrect. Conditions like Criggler-Najjar, Dubin-Johnson, Rotor, and Gilbert syndrome are associated with hyperbilirubinemia and jaundice, but not associated with abdominal mass. Crigler-Najjar syndrome is divided into two types. Type 1 (CN1) is very severe, and affected individuals can die in childhood due to kernicterus, although with proper treatment, they may survive longer. Type 2 (CN2) is less severe. People with CN2 are less likely to develop kernicterus, and most affected individuals survive into adulthood. Mutations in the UGT1A1 gene cause Crigler-Najjar syndrome. This gene provides instructions for making the bilirubin uridine diphosphate glucuronosyl transferase (bilirubin-UGT) enzyme, which is found primarily in liver cells and is necessary for the removal of bilirubin from the body. The bilirubin-UGT enzyme performs a chemical reaction called glucuronidation. During this reaction, the enzyme transfers a compound called glucuronic acid to unconjugated bilirubin, converting it to conjugated bilirubin. Glucuronidation makes bilirubin dissolvable in water so that it can be removed from the body. Mutations in the UGT1A1 gene that cause Crigler-Najjar syndrome result in reduced or absent function of the bilirubin-UGT enzyme. People with CN1 have no enzyme function, while people with CN2 have less than 20 percent of normal function. Individuals with Dubin-Johnson syndrome develop jaundice during adolescence or early adulthood. Jaundice is typically the only symptom. Dubin-Johnson syndrome is caused by mutations in the ABCC2 gene. The ABCC2 gene codes for a protein called multidrug resistance protein 2 (MRP2). This protein acts as a pump to transport bilirubin out of liver cells and into bile. Rotor syndrome is a relatively mild condition. Jaundice is usually evident shortly after birth or in childhood and may come and go. People with Rotor syndrome have a buildup of both unconjugated and conjugated bilirubin in their blood, but the majority is conjugated. The SLCO1B1 and SLCO1B3 genes are involved in Rotor syndrome. Mutations in both genes are required for the condition to occur. The SLCO1B1 and SLCO1B3 genes provide instructions for making similar proteins, called organic anion transporting polypeptide 1B1 (OATP1B1) and organic anion transporting polypeptide 1B3 (OATP1B3), respectively. Both proteins are found in liver cells; they transport bilirubin and other compounds from the blood into the liver so that they can be cleared from the body. Gilbert syndrome is a relatively mild condition characterized by periods of elevated levels of unconjugated bilirubin. In affected individuals, bilirubin levels fluctuate and very rarely increase to levels that cause jaundice. Gilbert syndrome is usually recognized in adolescence. If people with this condition have episodes of hyperbilirubinemia, these episodes are generally mild and typically occur when the body is under stress, for instance because of dehydration, prolonged periods without food (fasting), illness, vigorous exercise, or menstruation. Approximately 30 percent of people with Gilbert syndrome have no signs or symptoms of the condition and are discovered only when routine blood tests reveal elevated unconjugated bilirubin levels. Changes in the UGT1A1 gene cause Gilbert syndrome. This gene provides instructions for making the bilirubin uridine diphosphate glucuronosyltransferase (bilirubin-UGT) enzyme. Answer E is incorrect. Physiologic jaundice occurs in the first week of life and is not associated with abdominal mass. Answer F is incorrect. Drugs and toxins can cause hepatitis and cholestasis, but they would not result in an abdominal mass.

A previously healthy 17-year-old girl comes to the emergency department because of a 5-day history of progressive lower abdominal pain, fever, and malodorous vaginal discharge. Menarche was at the age of 12 years, and her last menstrual period was 2 weeks ago. She is sexually active with one male partner and uses a combination contraceptive patch. Her temperature is 37.8°C (100°F), pulse is 90/min, respirations are 22/min, and blood pressure is 110/70 mm Hg. Abdominal examination shows severe lower quadrant tenderness bilaterally. Pelvic examination shows a purulent cervical discharge, cervical motion tenderness, and bilateral adnexal tenderness. Her hemoglobin concentration is 10.5 g/dL, leukocyte count is 13,000/mm3, and platelet count is 345,000/mm3. A urine pregnancy test is negative. Which of the following is the most appropriate pharmacotherapy? (A) Oral azithromycin (B) Vaginal clindamycin (C) Intravenous penicillin and vancomycin (D) Intramuscular ceftriaxone and oral doxycycline (E) Intravenous oxacillin and metronidazole

Answer D is correct. The presence of purulent cervical discharge, cervical motion tenderness, and bilateral adnexal tenderness in a patient with fever and leukocytosis suggests a diagnosis of pelvic inflammatory disease. In a sexually active woman, Neisseria gonorrhea and Chlamydia trachomatis are likely pathogens and can occur concurrently. Ceftriaxone is effective treatment for Neisseria gonorrhea and doxycycline is effective treatment for Chlamydia trachomatis.

A 57-year-old man comes to the emergency department because of cramping in his hands and feet and numbness and tingling around his lips and in his fingers; these symptoms occurred intermittently for 6 months but have been progressively severe during the past 2 weeks. He also has had a 13-kg (30-lb) weight loss and bulky, foul-smelling stools that do not flush easily. He has a 10-year history of drinking 8 to 10 beers daily. He has been hospitalized twice for severe abdominal pain 4 and 6 years ago. His pulse is 80/min, and blood pressure is 105/65 mm Hg. He appears cachectic and chronically ill. The abdomen is nontender. Deep tendon reflexes are 4+ bilaterally. Chvostek and Trousseau signs are present. His serum calcium concentration is 6.5 mg/dL. Which of the following is the most likely diagnosis? A. Hypomagnesemia B. Hypoparathyroidism C. Osteomalacia D. Vitamin D deficiency

Answer D is correct. Tingling, cramping, hyperreflexia, along with a positive Chvostek and Trousseau sign indicate hypocalcemia, which is confirmed by laboratory testing. Malabsorption of vitamin D is the most likely cause, given the history of abdominal pain, weight loss, and bulky, foul-smelling stools. Hypomagnesemia is also quite possible in a patient with high alcohol consumption and malabsorption, but perhaps less likely than vitamin D deficiency. Fat-soluble vitamins (A, D, E, and K) are particularly prone to becoming deficient in malabsorption syndromes.

A 37-year-old woman, gravida 5, para 4, at 34 weeks' gestation comes to the emergency department because of vaginal bleeding for 2 hours. She has had no prenatal care. Her second child was delivered by lower segment transverse cesarean section because of a nonreassuring fetal heart rate; her other three children were delivered vaginally. Her pulse is 92/min, respirations are 18/min, and blood pressure is 134/76 mm Hg. The abdomen is nontender, and no contractions are felt. There is blood on the vulva, the introitus, and on the medial aspect of each thigh. The fetus is in a transverse lie presentation. The fetal heart rate is 144/min. Which of the following is the most likely diagnosis? A. Abruptio placentae B. Amniotic fluid embolism C. Latent phase of labor D. Placenta previa E. Ruptured uterus F. Ruptured vasa previa

Answer D is correct. Women with placenta previa often present with painless, bright red vaginal bleeding. This commonly occurs around 32 weeks of gestation, but can be as early as late mid-trimester. Answer A is incorrect. An acute, overt abruption typically presents with vaginal bleeding, abdominal pain and tenderness, and continuous uterine contractions. Answer B is incorrect. Amniotic fluid embolism presents during labor with shortness of breath, hypoxemia, and cardiovascular collapse. Answer C is incorrect. The initial phase of labor is termed the latent phase. It begins at the point at which the woman perceives regular uterine contractions. These contractions gradually soften, efface, and begin to dilate the cervix. The end of the latent phase occurs when the active phase begins. The active phase is defined as the period when changes in cervical dilation accelerate to at least 1 to 2 cm per hour and the fetus descends into the birth canal. Answer E is incorrect. Uterine rupture in pregnancy is a rare and often catastrophic complication with a high incidence of fetal and maternal morbidity. The signs and symptoms of uterine rupture largely depend on the timing, site, and extent of the uterine defect. The classic signs and symptoms of uterine rupture are (1) fetal distress (as evidenced most often by abnormalities in fetal heart rate), (2) diminished baseline uterine pressure, (3) loss of uterine contractility, (4) abdominal pain, (5) recession of the presenting fetal part, (6) hemorrhage, and (7) shock. However, modern studies show that some of these signs and symptoms are rare and that many may not be reliably distinguished from their occurrences in other, more benign obstetric circumstances. Answer F is incorrect. The classic triad of the vasa previa is membrane rupture, painless vaginal bleeding and fetal bradycardia.

A hospitalized 57-year-old man has had severe progressive pain in his left knee since awakening 2 hours ago. He was admitted to the hospital 2 days ago for an acute myocardial infarction. Cardiac catheterization showed occlusion of the left anterior descending artery, and he underwent placement of a stent. Current medications include aspirin, metoprolol, lisinopril, simvastatin, clopidogrel, and heparin. Vital signs are within normal limits. Examination of the knee shows a large effusion. The knee is hot to touch and erythematous. He holds the knee in 30 degrees of flexion; the pain is exacerbated with further flexion or extension. Laboratory studies show: Hematocrit 40% Leukocyte count 13,000/mm3 Ca2+ 9.2 mg/dL Urea nitrogen 15 mg/dL Creatinine 1.0 mg/dL Albumin 3.6 g/dL An x-ray of the left knee shows calcification of the synovium. Which of the following is the most likely diagnosis? (A) Deep venous thrombosis (B) Gonorrhea (C) Gout (D) Hemarthrosis (E) Pseudogout (F) Septic arthritis

Answer E is correct. Acute inflammation of a knee in an older person with synovial calcification (on x-ray) suggests calcium pyrophosphate dehydrate (CPPD) deposition disease, also called pseudogout.

A 67-year-old woman comes to the physician because of easy bruising for 4 months. She has a history of lung cancer treated with radiation therapy 6 months ago. She has a 2-year history of hypertension treated with a thiazide diuretic and an angiotensin-converting enzyme (ACE) inhibitor. Examination, including neurologic examination, shows no abnormalities except for multiple ecchymoses. Her hemoglobin concentration is 13 g/dL, leukocyte count is 5000/mm3, and platelet count is 35,000/mm3. A serum antiplatelet antibody assay is negative. Which of the following is the most appropriate next step in diagnosis? (A) Bone scan (B) CT scan of the abdomen (C) CT scan of the chest (D) Bronchoscopy (E) Bone marrow aspiration

Answer E is correct. Bone marrow aspiration can be done to distinguish thrombocytopenia associated with decreased platelet production from thrombocytopenia associated with increased platelet destruction. Moreover, it can help differentiate among the various types of hypoproliferative thrombocytopenia. In the vignette above, there are several possible explanations for thrombocytopenia, including bone marrow infiltration by cancer, thiazide-induced bone marrow suppression, and immune-mediated platelet destruction. If the bone marrow is infiltrated by cancer, malignant cells will appear in the aspirate. If thiazide-induced bone marrow suppression is present, megakaryocyte counts will be decreased, whereas in immune-mediated platelet destruction, megakaryocyte counts will be increased.

A 19-year-old man comes to the physician because of a 3-week history of malaise, generalized fatigue, swelling of his legs, and dark urine. He has no known sick contacts. There is no personal or family history of serious illness. He takes no medications. His temperature is 37°C (98.6°F), pulse is 82/min, respirations are 14/min, and blood pressure is 152/91 mm Hg. Examination shows 2+ pretibial edema bilaterally. The remainder of the examination shows no abnormalities. Laboratory studies show: Hemoglobin 10.4 g/dL Leukocyte count 5000/mm3 Platelet count 250,000/mm3 Serum Na+ 135 mEq/L K+ 4.9 mEq/L Cl− 101 mEq/L HCO3− 19 mEq/L Urea nitrogen 68 mg/dL Creatinine 4.6 mg/dL Urine Blood 3+ Protein 3+ RBC 5-7/hpf with dysmorphic features RBC casts numerous Serum complement concentrations are within the reference ranges. Renal ultrasonography shows no abnormalities. A renal biopsy specimen shows a crescent formation in the glomeruli and immune complex deposition along the basement membrane. The most appropriate next step in management is administration of which of the following? (A) Oral azathioprine (B) Oral lisinopril (C) Intravenous fluids (D) Intravenous furosemide (E) Intravenous methylprednisolone

Answer E is correct. Crescentic formation in the glomeruli of a patient with glomerulonephritis (red blood cell casts and dysmorphic red blood cells) and renal failure suggests RPGN. Intravenous glucocorticoids (e.g., methylprednisolone) are a mainstay of therapy.

A 9-month-old boy is brought to the physician because of discharge from the left ear for 1 day. He has had frequent infections since the age of 4 months. He had Streptococcus pneumoniae bacteremia at 4 months of age, Haemophilus influenzae meningitis at 5½ months of age, and S. pneumoniae pneumonia at 7 months of age. He also has had two episodes of otitis media during this period. He is an only child, and there is no family history of frequent infections. His immunizations are up-to-date. He is at the 20th percentile for length and 3rd percentile for weight. He is not in acute distress. Examination shows no abnormalities other than a purulent drainage from the left ear canal. Which of the following is the most likely diagnosis? A. Chronic granulomatous disease of childhood B. Severe combined immunodeficiency C. Thymic-parathyroid dysplasia (DiGeorge syndrome) D. Transient hypogammaglobulinemia of infancy E. X-linked agammaglobulinemia

Answer E is correct. Recurrent infections with encapsulated bacteria (Streptococcus pneumoniae and Haemophilus influenzae) despite adequate immunization suggest a defect in B-cell/immunoglobulin function, as seen in X-linked agammaglobulinemia. Answer A is incorrect. Patients with chronic granulomatous disease are predisposed to disease by catalase-producing organisms such as staphylococci, Serratia, Nocardia, and Aspergillus. Answer B is incorrect. SCID patients are usually affected by severe bacterial, viral, or fungal infections early in life and often present with interstitial lung disease, chronic diarrhea, and failure to thrive. Ear infections, recurrent Pneumocystis jirovecii pneumonia, and profuse oral candidiasis commonly occur. These babies, if untreated, usually die within 1 year due to severe, recurrent infections unless they have undergone successful hematopoietic stem cell transplantation. Answer C is incorrect. Children with DiGeorge syndrome suffer from recurrent viral and fungal infections within the first year of life. Answer D is incorrect. Infants with transient hypogammaglobulinemia of infancy (THI) typically begin to experience increasingly frequent and recurrent otitis media, sinusitis, and bronchial infections. Life-threatening infections with polysaccharide-encapsulated bacteria are unusual. Because antigen-specific antibody responses are largely intact, this likely accounts for the lack of serious bacterial infections observed in THI. In children older than 3 years, the frequency of infections typically diminishes, even if serum immunoglobulin levels have not yet normalized. T cell immunity is intact, and infections with opportunistic microorganisms do not usually occur.

A previously healthy 27-year-old man comes to the physician 4 weeks after noticing three nontender lesions on his penis. He says they have not changed in size. He is sexually active with multiple male and female partners and uses condoms inconsistently. He takes no medications. He drinks two to five beers on social occasions. He occasionally smokes marijuana. His temperature is 36.9°C (98.4°F). There is no lymphadenopathy. Examination shows three sessile, flesh-colored lesions on the shaft of the penis that are 10 mm in diameter. On application of a dilute solution of acetic acid, the lesions turn white. The remainder of the examination shows no abnormalities. Which of the following is the most appropriate next step in management? (A) Topical ganciclovir therapy (B) Oral acyclovir therapy (C) Oral doxycycline therapy (D) Intramuscular penicillin therapy (E) Cryotherapy

Answer E is correct. Cryotherapy can be used to treat genital warts. Answer A is incorrect. Ganciclovir is used to treat cytomegalovirus infection. Answer B is incorrect. Acyclovir is used to treat herpes simplex infection. Answer C is incorrect. Doxycycline is effective against Chlamydia. Answer D is incorrect. Penicillin is used to treat syphilis.

A 42-year-old woman comes to the physician because of a 1-year history of vaginal bleeding for 2 to 5 days every 2 weeks. The flow varies from light to heavy with passage of clots. Menses previously occurred at regular 25- to 29-day intervals and lasted for 5 days with normal flow. She has no history of serious illness and takes no medications. She is sexually active with one male partner, and they use condoms inconsistently. Her mother died of colon cancer, and her maternal grandmother died of breast cancer. She is 163 cm (5 ft 4 in) tall and weighs 77 kg (170 lb); BMI is 29 kg/m2. Her temperature is 36.6°C (97.8°F), pulse is 90/min, respirations are 12/min, and blood pressure is 100/60 mm Hg. The uterus is normal sized. The ovaries cannot be palpated. The remainder of the examination shows no abnormalities. Test of the stool for occult blood is negative. Which of the following is the most appropriate next step in diagnosis? (A) Barium enema (B) Progesterone challenge test (C) Colposcopy (D) Cystoscopy (E) Endometrial biopsy

Answer E is correct. In women with abnormal bleeding who are at risk for endometrial hyperplasia or endometrial cancer (including women older than 35 years, and women with obesity or a history of chronic anovulation), evaluation of the endometrium should be pursued. This can be done by office endometrial biopsy.

A 52-year-old woman comes to the emergency department because of a 1-week history of low-grade fever and increasing abdominal cramps that are exacerbated by bowel movements. She began a course of amoxicillin/clavulanate and metronidazole 2 days ago but has had no relief of her symptoms. She has had intermittent constipation for the past 12 years. She has not had nausea, vomiting, urinary symptoms, or bloody stools. She has a 3-year history of hypertension. She underwent total abdominal hysterectomy and bilateral salpingo-oophorectomy 5 years ago because of leiomyomata uteri. She is 165 cm (5 ft 5 in) tall and weighs 86 kg (190 lb); BMI is 32 kg/m2. Her temperature is 38.1°C (100.6°F), pulse is 75/min, and blood pressure is 150/80 mm Hg. The lungs are clear to auscultation. Cardiac examination shows no abnormalities. The abdomen is soft, and there is tenderness to palpation of the left lower quadrant with guarding but no rebound. Bowel sounds are normal. The stool is brown, and test for occult blood is negative. Her hemoglobin concentration is 14.5 g/dL, leukocyte count is 15,000/mm3, and platelet count is 280,000/mm3; serum studies and urinalysis show no abnormalities. Which of the following is the most appropriate next step in diagnosis? (A) Colonoscopy (B) Test of the stool for Clostridium difficile toxin (C) Endoscopic retrograde cholangiopancreatography (D) Pelvic ultrasonography (E) CT scan of the abdomen with contrast (F) Examination of the stool for ova and parasites

Answer E is correct. Left lower quadrant abdominal pain, associated with fever, constipation, and leukocytosis, suggests a diagnosis of diverticulitis. CT imaging is the test of choice. Answer A is incorrect. Flexible sigmoidoscopy allows visualization of the rectum, sigmoid colon, and descending colon to the level of the splenic flexure. Enemas are given before the procedure to clear stool from the distal colon. Because sigmoidoscopy is quick (<10 minutes) and not particularly painful, sedation is typically not provided, making it a convenient tool for colorectal cancer screening. Additional indications for sigmoidoscopy include acute and chronic diarrhea and rectal bleeding and for evaluating responses to therapies for colitis. Colonoscopy allows direct visualization of the entire large bowel and even several centimeters of terminal ileum. Bowel cleansing for colonoscopy requires the ingestion of osmotically active solutions, such as sodium phosphate or polyethylene glycol, coupled with a liquid diet for 24 hours before the procedure. Colonoscopy is more uncomfortable for the patient than sigmoidoscopy, which requires sedation. Indications for colonoscopy include those for sigmoidoscopy, as well as iron deficiency anemia, frank and occult GI blood loss, and assessing inflammatory bowel disease, including surveillance for dysplasia. Therapeutic interventions possible during colonoscopy include polypectomy, thermal ablation of vascular ectasias, decompression of colonic dilation associated with pseudo-obstruction, and occasionally the endoscopic control of lower GI bleeding. Colonoscopy should not be performed in a patient with acute diverticulitis, but is useful to visualize the colon after inflammation has subsided. Answer B is incorrect. C. difficile infection (CDI) is the most clinically significant cause of hospital-acquired diarrheal disease and is often accompanied by fever and leukocytosis. It is a cause of significant morbidity and mortality and becoming increasingly difficult to control and eradicate. An epidemic strain of C. difficile associated with more severe disease, mortality, and frequent relapses has emerged in the United States, Canada, and Europe since 2003. C. difficile now rivals MRSA as the most common organism to cause hospital-acquired infections in the United States. C. difficile infection (CDI) is a gastrointestinal infection characterized by diarrhea (three or more loose or unformed stools in ≤24 hours) and a positive test for C. difficile toxin A or toxin B in stool, evidence of a toxin-producing strain of C. difficile in stool, or evidence of pseudomembranous colitis on direct visualization of the colon. C. difficile is a spore-forming, anaerobic, gram-positive organism that survives well in water, soil, and animals and has a worldwide distribution. CDI occurs most frequently in health care settings, particularly in long-term care facilities and acute care hospitals, and is most frequent and lethal among the elderly, especially those older than 75 years. Rates in U.S. hospitals have nearly tripled in the decade since 2000, and it is now estimated that there are 500,000 to 750,000 CDIs each year. A specific strain of C. difficile—identified as the restriction endonuclease group BI, pulse field gel type NAP1, PCR ribotype 027 (BI/NAP1/027) strain—is thought to be responsible for much of the epidemic that has extended to Canada and Europe. Rates of CDI in the community have also increased somewhat during the past decade, in association with the rising rates in hospitals and nursing homes. However, it is unlikely that community cases account for more than 15 to 20% of all CDIs. Risk factors for CDI include antimicrobial use, advanced age, and stay in an acute or chronic care facility. Hospitals are considered a particularly high-risk environment because patients are elderly, antimicrobial use is frequent, the environment is contaminated with C. difficile spores (which are difficult to eradicate), asymptomatic patients carry C. difficile in their stools, and health care workers carry C. difficile on their hands if they do not practice good hand hygiene. In about 2 to 3% of healthy adults, C. difficile can be cultured from their stools, but the frequency in asymptomatic hospitalized patients increases with the duration of hospitalization and may reach 20% or more. Exposure to nearly all antimicrobials has been associated with subsequent CDI, but those with the highest risk are clindamycin, the cephalosporins, and the fluo¬roquinolones. CDI is rare in children and young adults, despite their frequent exposure to antimicrobials. However, children younger than 1 year are com¬monly colonized with C. difficile but remain asymptomatic, an observation that remains largely unexplained. CDI risk appears to be minimal in the absence of antimicrobial therapy. When antimicrobials are administered, they have the unintended conse¬quence of disrupting the normal protective bowel flora, which may persist for days to weeks after the antimicrobial is stopped. If C. difficile is ingested during this time, the spores germinate in the gut, and the vegetative form of the organism multiplies and begins to make toxins. At this point, whether the patient will develop diarrhea is dictated by the status of his or her immunity to the toxins, which is best correlated with serum immunoglobulin G (IgG) antibodies directed at toxin A and toxin B of C. difficile. Those with good antibody responses will be asymptomatic but will remain colonized with C. difficile, whereas those with little or no antibody response will develop diar¬rhea and CDI. Toxin A is primarily an enterotoxin, and toxin B is a cytotoxin. Both act by glucosylation that disrupts the cell cytoskeleton, resulting in colonic epithelial cell rounding, fluid leakage, and cell death. In the presence of pseudomembranous colitis, the colon appears to be covered in yellow to white pseudomembranes that vary in size from punctate to completely con¬fluent and covering the entire colon in advanced cases. Histologically, the colon demonstrates a marked neutrophil infiltration throughout the colon wall, with mucosal necrosis and volcano-like lesions from which the pseudo¬membrane is seen to "erupt." The pseudomembrane is composed of protein¬aceous material and cellular debris. Clinical symptoms of CDI range from asymptomatic carriage to severe and sometimes life-threatening pseudomembranous colitis complicated by major fluid losses and systemic complications. With mild CDI, patients may simply have "nuisance diarrhea" that resolves when the implicated drug is discontin¬ued. Others with more severe CDI have substantial fluid and protein losses combined with fever, cramps, hypoalbuminemia, leukocytosis, and hypoten¬sion. Leukocytosis is common (occurring in up to 50% of patients) and is a marker of severe CDI when it exceeds 15,000/mm3. Extremely high levels greater than 50,000/mm3 are an indication of fulminant and potentially fatal illness. Other factors that may be indicative of severe or late-stage disease include toxic megacolon, high fever, renal failure, hypotension, and a lactic acidosis greater than 5.0 mmol/L. The diagnosis should be suspected in any patient who has otherwise unex¬plained diarrhea (three or more loose or unformed stools in ≤24 hours) in association with recent or concurrent antibiotic use. Only about 10 to 20% of patients in this category actually have CDI, but this is the patient group that should be tested. The standard test to establish the diagnosis is to detect toxins A and B in stool or to detect a toxin-producing strain of C. difficile in stool. The cause of diarrhea in an adult patient with an onset longer than 48 hours after hospital admission is almost always CDI because other infectious enteric pathogens are extremely rare in the hospital setting. Also common in the differential diagnosis are noninfectious causes of diar¬rhea, such as antibiotic or other drug-associated diarrhea, ischemic colitis, and inflammatory bowel disease. Two major prevention strategies are employed. The first is traditional infec¬tion control in which barriers to transmission (gowns, gloves, isolation, hand hygiene, environmental cleaning) are used to prevent the spores of C. difficile from reaching the patient. The second strategy is to reduce the likelihood of infection if the patient does encounter C. difficile while in the hospital. The most efficacious strategy, also known as antimicrobial stewardship, is to avoid or minimize exposure to antimicrobials, especially those with a high CDI risk such as clindamycin, cephalosporins, and fluoroquinolones. Interventions to restrict exposure to clindamycin and to cephalosporins have been highly effective in interrupting outbreaks of CDI in hospitals. Treatment of CDI begins with discontinuation of the implicated antibiotic, supportive care, and avoidance of antiperistaltic agents. Mildly ill patients may recover with these simple conservative measures, but most require specific treatment. Metronidazole, at a dose of 500 mg orally three times a day for 10 to 14 days, is the recommended treatment for patients with mild CDI. It provides response rates comparable to those for vancomycin, is much less expensive, and avoids some of the concern about vancomycin-resistant enterococcus colonization. Patients with severe CDI (variously defined as a white blood cell count >15,000 or a creatinine increase to >1.5-fold over base¬line) should be treated with vancomycin 125 mg orally four times a day for 10 days. The anticipated response to these drugs is rapid defervescence, with gradual normalization of bowel habits. Mean time to resolution of diarrhea is about 3 days; if symptoms have not resolved by day 5 or 6 of treatment, a change in therapy should be con¬sidered. However, there are no data to support the use of more than one drug at a time to treat CDI. Failure to respond often means that either the disease has progressed too far or another condition is causing the symptoms. In this patient, a diagnosis of C. difficile colitis is unlikely given the absence of diarrhea. Answer C is incorrect. Endoscopic retrograde cholangiopancreatography (ERCP) is a combined endoscopic and radiographic procedure for imaging the biliary and pancreatic ducts. A duodenoscope is a specially designed instrument for use during ERCP that includes an imaging lens oriented on the side of the endoscope's tip, allowing a direct view of the ampulla of Vater on the medial wall of the second portion of the duodenum. A tiny finger-like projection called an elevator helps the endoscopist guide a catheter into the duct of interest. Contrast is then injected through the catheter, filling the duct, and fluoroscopic images are obtained. ERCP is indicated for evaluating obstructive jaundice with or without suppurative cholangitis, biliary colic with suspected bile duct stones, chronic or recurrent acute pancreatitis, and suspected primary sclerosing cholangitis. Bile duct brushings and even biopsies may be obtained to determine whether a biliary stricture is neoplastic. With the use of a special manometry catheter, sphincter of Oddi pressures can also be measured in cases of suspected sphincter of Oddi dysfunction. Therapeutic interventions possible during ERCP include sphincterotomy (an incision through the sphincter of Oddi using a catheter with an electrocautery cutting wire), removal of bile duct stones, and placement of biliary or pancreatic duct stents to alleviate signs and symptoms of obstruction. ERCP carries a significant (5%) risk for complications, including pancreatitis, post-sphincterotomy bleeding, and perforation. In this patient, there is no evidence of biliary disease, and so ERCP would not be useful. Answer D is incorrect. A pelvic ultrasound is a noninvasive procedure used to assess organs and structures within the female pelvis. A pelvic ultrasound allows quick visualization of the female pelvic organs and structures including the uterus, cervix, vagina, fallopian tubes, and ovaries. Doppler ultrasound may also show blood flow in certain pelvic organs. The patient in this vignette has undergone total abdominal hysterectomy and bilateral salpingo-oophorectomy. Pelvic ultrasound would not be useful. Answer F is incorrect. Examination of the stool for ova and parasites is performed when there is suspicion of intestinal infection with Giardia lamblia or Entamoeba histolytica. Persistent traveler's diarrhea that is unresponsive to empiric antibiotics is often caused by Giardia lamblia, transmitted by contaminated water or food. Three stool specimens for ova and parasites and a stool culture are warranted. Unfortunately, G. lamblia may be missed in up to one third of patients even after this workup. Assessment of stool Giardia antigen by enzyme-linked immunosorbent assay is more sensitive than culture. If clinical suggestion is high, an empiric course of metronidazole (500 mg orally three times a day for 7 days) is usually justified. Giardiasis is endemic to the United States and is also a common cause of persistent (longer than 2 to 3 weeks), nonbloody diarrhea in returning travelers. G. lamblia is prevalent throughout much of the developing world. Latin America is the most common site of Giardia acquisition by North American travelers. Men who have sex with men also have a high prevalence of infection because of specific sexual practices. Persons who work in day-care centers are also at increased risk for giardiasis because of its relatively high prevalence in young children. As with Giardia, Entamoeba histolytica is transmitted by the fecal-oral route; the vast majority of infected individuals are asymptomatic. When E. histolytica causes acute illness, it is generally identified by bloody diarrhea. In the United States, amebic dysentery is occasionally misdiagnosed as ulcerative colitis or Crohn disease, and the administration of corticosteroids may cause significant worsening and toxic megacolon. Stool examination for ova and parasites is generally diagnostic, but sigmoidoscopy may be required. Serologic testing is useful to exclude amebiasis in individuals from industrialized countries because the background antibody positivity is quite low. Treatment involves a 5- to 10-day course of metronidazole. Extraintestinal amebiasis generally manifests as hepatic liver abscess. The patient in this vignette does not have a diarrheal illness. Examination of stool for ova and parasites would not be helpful.

Two days after admission to the hospital for congestive heart failure, an 82-year-old man is unable to walk because of severe, throbbing pain in his left foot. He has no history of similar episodes or recent trauma. He also has coronary artery disease and hypertension. Current medications include atenolol, lisinopril, furosemide, and aspirin. He does not smoke or drink alcohol. He is in moderate distress. His temperature is 38°C (100.4°F), pulse is 68/min and regular, respirations are 12/min, and blood pressure is 138/88 mm Hg. Jugular venous pulsations are present 3 cm above the sternal angle. Crackles are heard at both lung bases. A grade 2/6 systolic murmur is heard best at the left sternal border and second intercostal space. Examination of the lower extremities shows pitting pedal edema. There is tenderness, erythema, and edema of the left great toe. Active and passive range of motion of the first metacarpophalangeal joint produces pain; arthrocentesis of the joint is performed. Analysis of joint fluid aspirate is most likely to show which of the following? WBC Microscopic Examination for Crystals Gram Stain (/mm3) (A) 100 needle-shaped no organisms (B) 100 none gram-positive cocci (C) 100 none no organisms (D) 100 rhomboid no organisms (E) 20,000 needle-shaped no organisms (F) 20,000 none gram-positive cocci

Answer E is correct. Pain, erythema, and swelling of the big toe in a patient who is on medicines that raise serum uric acid levels (aspirin, furosemide) suggest a diagnosis of gout, presenting as podagra. The synovial fluid of a patient with gout is highly inflammatory (PMNs > 10,000) and contains needle-shaped, negatively birefringent crystals, but no organisms.

A 25-year-old woman comes to the physician because of a 2-month history of numbness in her right hand. During this period, she has had tingling in the right ring and small fingers most of the time. She has no history of serious illness and takes no medications. She is employed as a cashier and uses a computer at home. She played as a pitcher in a softball league for 5 years until she stopped 2 years ago. Vital signs are within normal limits. Examination shows full muscle strength. Palpation of the right elbow produces a jolt of severe pain in the right ring and small fingers. Sensation to pinprick and light touch is decreased over the medial half of the right ring finger and the entire small finger. The most likely cause of these findings is entrapment of which of the following on the right? (A) Brachial plexus at the axilla (B) Median nerve at the wrist (C) Musculocutaneous nerve at the forearm (D) Radial nerve at the forearm (E) Ulnar nerve at the elbow

Answer E is correct. Pain, tingling, and loss of sensation over the medial half of the right ring finger and the entire small finger indicate entrapment of the right ulnar nerve. The elbow is the most common location for ulnar nerve entrapment. Moreover, palpation of the right elbow produces a jolt of severe pain in the ulnar distribution.

A 67-year-old woman comes to the physician 1 month after noticing a nontender nodule on the back of her left hand. She initially thought it was an insect bite, but it has grown in size over the past week. It bleeds when she picks at it. She has no history of serious illness. She lives in a retirement community in Florida and frequently plays golf and tennis. Examination of the dorsum of the right hand shows a 2.5-cm lesion. A photograph of the lesion is shown. Which of the following is the most appropriate next step in management? (A) Observation (B) Topical application of fluorouracil (C) Sentinel lymph node biopsy (D) Cryosurgery (E) Excision of the lesion

Answer E is correct. Squamous cell carcinoma (SCC) is commonly a red, scaling, thickened patch on sun-exposed skin. Some are firm hard nodules and dome-shaped like keratoacanthomas. Ulceration and bleeding may occur. When SCC is not treated, it may develop into a large mass. Excision is the mainstay of therapy . Answer A is incorrect. Observation is not a good idea. The cancer will continue to grow. Answer B is incorrect. Topical application of fluorouracil can be used for treating actinic (solar) keratoses and some types of basal cell carcinomas. Answer C is incorrect. Sentinal lymph node biopsy is most commonly used to help stage breast cancer and melanoma. However, it is being studied with other cancer types, including colorectal cancer, gastric cancer, esophageal cancer, head and neck cancer, thyroid cancer, and non-small cell lung cancer. Answer D is incorrect. Warts, moles, skin tags, solar keratoses, Morton's neuroma and very small skin cancers are candidates for cryosurgical treatment. A common method of freezing lesions is using liquid nitrogen as the cooling solution.

A 37-year-old woman comes to the physician because of shortness of breath for 3 months. Her symptoms increase with exertion. She also has had difficulty walking because of ankle pain. Two months ago, she had painful red spots on her legs for several weeks. She has not had a sore throat. She takes acetaminophen for aches and pains. Her temperature is 37.4°C (99.3°F), pulse is 90/min, respirations are 18/min, and blood pressure is 140/90 mm Hg. The lungs are clear to auscultation, and cardiac examination shows no abnormalities. An x-ray of the chest shows bilateral hilar adenopathy. Which of the following is the most likely diagnosis? (A) Cat-scratch disease (B) Chronic lymphocytic leukemia (C) Hodgkin disease (D) Rubella (E) Sarcoidosis (F) Syphilis (G) Systemic lupus erythematosus (H) Tuberculosis

Answer E is correct. The combination of dyspnea, arthropathy, erythema nodosum, and hilar lymphadenopathy suggests a diagnosis of sarcoidosis.

A 16-year-old girl is brought to the physician because of intermittent pain and swelling of both ankles over the past month. She is currently not in pain. When the pain occurs, it is so severe that she is unable to walk. There is no associated fever or chills. She is sexually active and has had one sexual partner for 12 months. Her temperature is 37°C (98.6°F), pulse is 80/min, and blood pressure is 145/87 mm Hg. Examination shows no abnormalities or tenderness of the ankle joints. There is a nonpainful ulcer on the oral buccal mucosa. The lungs are clear to auscultation. Cardiac examination shows no abnormalities. Laboratory studies show: Leukocyte count 4000/mm3 Segmented neutrophils 65% Eosinophils 3% Lymphocytes 25% Monocytes 7% Platelet count 60,000/mm3 ESR 100 mm/h Serum Antinuclear antibodies 1:320 Anti-DNA antibodies positive Rapid plasma reagin 1:16 Rheumatoid factor negative Urine Protein 3+ RBC casts negative RBC none WBC 10-20/hpf X-rays of the ankles show no abnormalities other than tissue swelling. Which of the following is the most likely diagnosis? (A) Disseminated gonococcal disease (B) Polyarticular arthritis (C) Reactive arthritis (D) Secondary syphilis (E) Systemic lupus erythematosus

Answer E is correct. The diagnosis of systemic lupus erythematosus is suggested by the presence of arthritis, painful oral ulcer, leukopenia, thrombocytopenia, renal disease (proteinuria), positive ANA, positive anti-native DNA, and positive RPR (false-positive).

A previously healthy 20-year-old college student comes to the emergency department because she has been unable to urinate for 8 hours. She has a 2-day history of fever, fatigue, severe burning with urination, and pain in the genital area. She is sexually active and uses a diaphragm for contraception. She takes no medications. She appears uncomfortable. Her temperature is 38.7°C (101.7°F), pulse is 110/min, and blood pressure is 110/70 mm Hg. Abdominal examination shows a large, cystic, midline pelvic mass. The groin nodes are enlarged and tender. Pelvic examination shows erythematous external genitalia with extensive ulceration. The lesions extend into the vagina. Internal examination is not possible due to patient discomfort. Which of the following is the most appropriate initial step in management? (A) Measurement of serum urea nitrogen concentration (B) Vaginal cultures (C) Intravenous pyelography (D) Renal ultrasonography (E) Placement of a Foley catheter

Answer E is correct. The patient in the vignette has urinary retention caused by vulvovaginitis. Urinary retention of any cause is an emergency because it leads to hydronephrosis and renal failure. The first priority is relieving the obstruction. This can be done by insertion of a Foley catheter. Other diagnostic procedures, such as measurement of blood urea nitrogen, vaginal cultures, and renal ultrasonography, are useful but should be postponed until urinary retention is relieved.

A 62-year-old white man comes to the physician because of an 8-month history of progressive pain and stiffness of his hands. The stiffness is worse at the end of the day. He has a 1-year history of fatigue and increased urination. He has no history of serious illness and takes no medications. His last visit to a physician was 10 years ago. He does not smoke or drink alcohol. He is 185 cm (6 ft 1 in) tall and weighs 82 kg (180 lb); BMI is 24 kg/m2. His pulse is 84/min, and blood pressure is 136/82 mm Hg. Examination shows dark brown skin. S1 and S2 are normal. An S3 is heard at the apex. There is mild tenderness over the second and third metacarpophalangeal joints bilaterally without synovial thickening. Heberden nodes are present over the distal interphalangeal joints of the index and ring fingers bilaterally. Laboratory studies show: Hemoglobin 16 g/dL Leukocyte count 7700/mm3 Platelet count 332,000/mm3 Glucose 182 mg/dL Albumin 3.4 g/dL Total bilirubin 1.1 mg/dL Alkaline phosphatase 52 U/L AST 55 U/L ALT 68 U/L Hepatitis B surface antigen negative Hepatitis C antibody negative Rheumatoid factor negative Which of the following is most likely to have prevented this patient's condition? (A) Calcium supplementation (B) Enalapril therapy (C) Metformin therapy (D) Methotrexate therapy (E) Phlebotomy

Answer E is correct. The triad of dark skin, diabetes mellitus, and abnormal liver enzymes suggest a diagnosis hemochromatosis. This condition also explains the fatigue, arthralgias, and cardiomyopathy seen in this patient. Joint pain typically affects the 2nd and 3rd metacarpophalangeal joints. Treatment is phlebotomy, with a goal of maintaining ferritin below 50 ng/mL and transferrin saturation below 30%.

A 62-year-old woman comes to the physician for a routine health maintenance examination. On questioning, she has had fatigue, constipation, and a 9-kg (20-lb) weight gain during the past year. She receives estrogen replacement therapy. Serum lipid studies were within the reference range 5 years ago. She is 157 cm (5 ft 2 in) tall and weighs 77 kg (170 lb); BMI is 31 kg/m2. Physical examination shows no other abnormalities. Serum lipid studies today show: Total cholesterol 269 mg/dL HDL-cholesterol 48 mg/dL LDL-cholesterol 185 mg/dL Triglycerides 180 mg/dL Which of the following is the most likely cause? (A) Alcohol (B) Diabetes mellitus (C) Estrogen deficiency (D) Estrogen replacement therapy (E) Hypothyroidism (F) Thiazide diuretic therapy

Answer E is correct. This first step in evaluating a patient with dyslipidemia is excluding secondary causes, such as diabetes, obesity, hypothyroidism, estrogen use, glucocorticoid use, alcohol consumption, and nephrotic syndrome. In a dyslipidemic patient with fatigue, constipation, and weight gain, hypothyroidism is a likely explanation for the abnormal lipid levels. Treated the hypothyroidism will improve lipid levels.

A previously healthy 34-year-old woman is brought to the physician because of fever and headache for 1 week. She has not been exposed to any disease. She takes no medications. Her temperature is 39.3°C (102.8°F), pulse is 104/min, respirations are 24/min, and blood pressure is 135/88 mm Hg. She is confused and oriented only to person. Examination shows jaundice of the skin and conjunctivae. There are a few scattered petechiae over the trunk and back. There is no lymphadenopathy. Physical and neurologic examinations show no other abnormalities. Test of the stool for occult blood is positive. Laboratory studies show: Hematocrit 32% with fragmented and nucleated erythrocytes Leukocyte count 12,500/mm3 Platelet count 20,000/mm3 Prothrombin time 10 sec Partial thromboplastin time 30 sec Fibrin split products negative Urea nitrogen 35 mg/dL Creatinine 3.0 mg/dL Bilirubin Total 3.0 mg/dL Direct 0.5 mg/dL Lactate dehydrogenase 1000 U/L Blood and urine cultures are negative. A CT scan of the head shows no abnormalities. Which of the following is the most likely diagnosis? (A) Disseminated intravascular coagulation (B) Immune thrombocytopenic purpura (C) Sarcoidosis (D) Systemic lupus erythematosus (E) Thrombotic thrombocytopenic purpura

Answer E is correct. This patient has the classic pentad of thrombotic thrombocytopenic purpura (TTP): fever, thrombocytopenia, microangiopathic hemolysis, neurologic symptoms, and renal insufficiency. Neurologic symptoms include confusion and disorientation. Several findings suggest hemolysis. The presence of nucleated erythrocytes on peripheral blood smear indicates that the bone marrow is "churning out" reticulocytes to compensate for cell loss. Also, the serum bilirubin level is elevated, and most of it is unconjugated (i.e., indirect). By contrast, liver disease predominantly raises conjugated, or direct, bilirubin. Many conditions raise lactate dehydrogenase levels (LDH), including hemolysis, which is often associated with very high LDH levels. This patient's hemolysis is microangiopathic as revealed by the presence of fragmented red blood cells (schistocytes). Answer A is incorrect. A life-threatening causes of nonimmune platelet destruction is disseminated intravascular coagulation (DIC), which is associated with sepsis, malignancy, advanced liver disease, and other disorders that trigger endotoxin release or cause severe tissue damage. In DIC caused by bacterial sepsis, circulating endotoxin induces expression of tissue factor on circulating monocytes and endothelial cells, a process leading to overwhelming thrombin and fibrin generation. Deposition of fibrin occurs throughout the vasculature, with relatively inadequate concurrent fibrinolysis leading to a thrombotic or microangiopathic vasculopathy and subsequent organ damage. Thrombin activation of platelets and circulating factors eventually overwhelms the bone marrow and liver synthetic capability, respectively, resulting in thrombocytopenia and prolongation of the PT and PTT. Thus, although the primary lesion of DIC is thrombin and clot generation, the clinical endpoint is usually a consumptive coagulopathy with depletion of platelets and coagulation factors. Mucosal bleeding, especially in the GI tract, and oozing from intravenous puncture sites are early signs of DIC. Fibrinogen levels are usually low in DIC. Fibrinolysis is triggered by fibrin clot formation and the action of tissue-type plasminogen activator; laboratory testing shows increased levels of fibrin split products and D-dimer (cleavage of fibrin-fibrin bonds). The blood smear may also help in the diagnosis of DIC by showing significant numbers of schistocytes; however, this finding is not specific to DIC and is present in other microangiopathies such as TTP. In this patient, the absence of fibrin split products and the normal prothrombin time (PT) and partial thromboplastin time (PTT) argue against the diagnosis of DIC. Answer B is incorrect. In children, acute immune thrombocytopenic purpura (ITP) is often preceded by a viral infection, such as varicella. Patients with ITP exhibit petechial hemorrhage and mucosal bleeding; platelet counts are often lower than 20,000/mcL. The blood smear shows large platelets but no other abnormal cells such as blasts, which would accompany childhood leukemia. The bone marrow demonstrates increased or occasionally normal numbers of megakaryocytes. The diagnosis of ITP is partly made by exclusion. Fever, organomegaly, pancytopenia, lymphadenopathy, or abnormal peripheral blood cells should prompt an evaluation for malignant disease, such as leukemia, neuroblastoma, or Wilms tumor, or other bone marrow disorders. Laboratory tests may complement the clinical evaluation but are not required to make the diagnosis of ITP. These tests include the demonstration of an increased percentage of reticulated platelets in the peripheral blood or the detection of platelet autoantibodies in serum or on the platelet (platelet-associated immunoglobulin). However, assays of platelet-associated antibodies, though sensitive, are not specific for ITP. In contrast, techniques that measure serum antibodies to specific platelet glycoproteins have greater specificity but are relatively insensitive. An increase in the reticulated platelet percentage is consistent with increased platelet destruction but cannot distinguish between ITP and other causes of platelet destruction. Thus the diagnosis of ITP remains largely clinical and does not require laboratory confirmation. Acute ITP in children can resolve without therapy, but most clinicians prefer to treat children with steroids or intravenous immunoglobulin (IVIG). Acute ITP in adults rarely remits spontaneously and is more likely to become a chronic disorder. Petechial hemorrhage and mucosal bleeding are accompanied by platelet counts commonly lower than 20,000/mcL and often as low as 1000 to 2000/mcL. In adults, ITP may be associated with other diseases, such as HIV or hepatitis C infection. ITP may be the presenting manifestation of HIV infection, whereas thrombocytopenia in more advanced stages of HIV infection is more often caused by bone marrow failure resulting from megakaryocyte infection with HIV, mycobacterial infection of the bone marrow, and nutritional deficiencies of end-stage HIV disease. ITP also occurs in patients with autoimmune disorders such as systemic lupus erythematosus, inflammatory bowel disease, and nonviral hepatitis. When the lupus anticoagulant or anticardiolipin antibody is present in association with systemic lupus erythematosus and thrombocytopenia, the diagnosis of secondary antiphospholipid antibody syndrome is made; this entity is most commonly associated with thromboembolic complications. The first-line treatment of acute ITP in adults is steroids, usually prednisone. For all chronic ITP patients, more than 50% have some degree of disease remission after splenectomy. ITP is not associated with microangiopathic hemolytic anemia or renal failure, which occur in this patient. Answer C is incorrect. Sarcoidosis is not typically associated with thrombocytopenia. Answer D is incorrect. Although systemic lupus erythematosus can be associated with hemolytic anemia, thrombocytopenia, confusion, and renal failure, as seen in this patient, lupus is not associated with microangiopathic hemolysis. The presence of fragmented red blood cells in this case argues against the diagnosis of lupus.

A 57-year-old woman comes to the physician because of an 8-week history of difficulty sleeping, fatigue, and muscle tension. During this period, she also has had memory lapses, difficulty concentrating, and has been reprimanded at work for arriving late. Over the past 2 weeks, she has had three episodes of palpitations and shortness of breath that have awakened her from sleep. Her pulse is 80/min, and blood pressure is 110/90 mm Hg. Physical examination shows no abnormalities. Mental status examination shows a depressed mood and constricted affect. She says that she is no longer interested in activities that she used to enjoy. She has suicidal ideation without a plan. Her hemoglobin concentration is 11 g/dL, and serum ferritin concentration is 140 ng/mL. Which of the following is the most appropriate initial step in treatment? (A) Acupuncture (B) Diazepam therapy (C) Donepezil therapy (D) Ferrous sulfate therapy (E) Ginkgo biloba extract therapy (F) Citalopram therapy

Answer F is correct. A diagnosis of depression is suggested by a constellation of symptoms, including fatigue, sleep disturbance, loss of interest in activities, difficulty concentrating, and suicidal ideation. SSRIs (selective serotonin reuptake inhibitors), such as citalopram, are effective for depression. Answer A is incorrect. Acupuncture is a collection of procedures which involves the stimulation of points on the body using a variety of techniques, such as penetrating the skin with needles that are then manipulated manually or by electrical stimulation. It is one component of traditional Chinese medicine (TCM) and is among the oldest healing practices in the world. According to the traditional Chinese medicine approach, stimulating specific acupuncture points corrects imbalances in the flow of qi through channels known as meridians. Scientific investigation has not found any histological or physiological correlates for traditional Chinese concepts such as qi, meridians, and acupuncture points. Research suggests that traditional forms of acupuncture are more effective than placebos in the relief of certain types of pain and post-operative nausea. Recent systematic reviews found that acupuncture also seems to be a promising treatment option for anxiety, sleep disturbances, and depression, but that further research is needed in these regards. Answer B is incorrect. Diazepam, a benzodiazepine, is an anxiolytic, not an anti-depressant. Many patients, such as the one described above, have anxiety as part of their depression. Treatment of the anxiety alone (with a benzodiazepine) can worsen depression. If depression lies at the root of a patient's anxiety, treating the depression may improve the anxiety. Answer C is incorrect. Donepezil is used to treat dementia, not depression. Depression can be confused with Alzheimer dementia because depression can be associated with memory loss (as in the patient above). The term "pseudo-dementia" is used to describe depressed patients whose symptoms mimic those of patients with dementia. Answer D is incorrect. Ferrous sulfate is used to treat iron-deficiency anemia. The patient above has a normal serum ferritin level, which rules out iron deficiency. Answer E is incorrect. Ginkgo biloba has been studied as a possible treatment for Alzheimer disease. Preliminary studies were encouraging, but the current understanding is that Ginkgo is not an effective treatment or preventative for dementia.

A 33-year-old woman comes to the emergency department 30 minutes after the sudden onset of chest pain, palpitations, shortness of breath, numbness and tingling in both arms, and fear of going crazy. She has visited local emergency departments several times over the past 3 months for similar symptoms that resolved within 1 hour. She uses an oral contraceptive. She drinks two beers daily and six beers on the weekend. She has no history of medical problems. Her mother and sisters have a history of anxiety. Her pulse is 90/min, respirations are 18/min, and blood pressure is 130/90 mm Hg. Physical examination, laboratory studies, and an ECG show no abnormalities. Which of the following is the most likely cause of anxiety? (A) Alcohol withdrawal (B) Coronary insufficiency (C) Hyperthyroidism (D) Hypoglycemia (E) Major depressive disorder (F) Panic disorder (G) Paroxysmal atrial tachycardia (H) Pheochromocytoma (I) Post-traumatic stress disorder (J) Pulmonary embolus (K) Somatization disorder

Answer F is correct. Repeated sudden episodes of chest pain, palpitations, shortness of breath, numbness and tingling, and fear in an otherwise young healthy woman suggest a diagnosis of panic disorder.

Six hours after delivery, a 1200-g (2-lb 11-oz) newborn develops respiratory distress. She was born at 32 weeks' gestation. Her pulse is 136/min, respirations are 60/min, and blood pressure is 60/30 mm Hg. Examination shows grunting and moderate intercostal and subcostal retractions. The lungs are clear to auscultation. Umbilical artery blood gas analysis on 60% oxygen shows: pH 7.32 PCO2 32 mm Hg PO2 60 mm Hg An x-ray of the chest shows diffuse reticulogranular densities with an air bronchogram. Which of the following is the most likely underlying mechanism? (A) Abnormality of mucociliary function (B) Aspiration of meconium into the lungs (C) Blockage of airways with mucus (D) Increased pulmonary capillary permeability (E) Infection with group B streptococcus (F) Pulmonary surfactant deficiency

Answer F is correct. Respiratory distress syndrome occurs primarily in premature infants. It occurs in 15-30% of those between 32 and 36 weeks gestation. Manifestations include respiratory distress, tachypnea, tachycardia, hypotension, hypoxemia, and reticulogranular densities on chest x-ray. The condition is caused by deficiency of surfactant. Other conditions in the differential diagnosis - early-onset sepsis with Group B Streptococcus, pneumonia, and meconium aspiration - can have a similar presentation but are less common.

A 5-year-old boy is brought to the physician because of a 1-week history of excessive bruising and bleeding from minor cuts. He also has had a 2-week history of pain in his extremities. He appears pale. His temperature is 38.3°C (100.9°F). Examination shows bleeding gums and scattered petechiae and purpura over the trunk and extremities. There is generalized lymphadenopathy and splenomegaly. What is the most likely cause of bleeding? A. Factor VIII deficiency B. Factor IX deficiency C. Fibrinogen deficiency D. Protein C deficiency E. Protein S deficiency F. Thrombocytopenia G. Vitamin K deficiency H. von Willebrand factor deficiency

Answer F is correct. Small vessel bleeding in the skin, in the mucous membranes, or in the gastrointestinal (GI) tract tends to occur more often in patients with thrombocytopenia, qualitative platelet defects, vascular abnormalities, and von Willebrand disease (vWD). Large vessel bleeding in organs, joints, or muscles is more commonly associated with factor deficiencies, such as hemophilia. The patient in this vignette has small vessel bleeding. The presence of petechiae suggests thrombocytopenia as a likely cause. In a child with lymphadenopathy and splenomegaly, this could indicate acute lymphoblastic leukemia.

A 64-year-old woman comes to the physician because of a 5-month history of increasing shortness of breath, sore throat, and a cough productive of a small amount of white phlegm. Over the past week, she has had nausea related to excess coughing. Over the past year, she has had a 3.2-kg (7-lb) weight loss. She has asthma treated with theophylline and inhaled β-adrenergic agonists and corticosteroids. She has smoked one pack of cigarettes daily for 44 years and drinks one alcoholic beverage daily. She appears thin. Examination shows a 2-cm, nontender lymph node in the right supraclavicular area. Examination shows no other abnormalities. An x-ray of the chest shows a large right lower lobe density. A CT scan of the chest shows a 7.5 x 7.5 x 6-cm right lower lobe mass with some scattered calcifications. The lesion abuts the posterior chest wall without clear invasion. There are right lower peritracheal, precarinal, right hilar, and subcarinal lymph nodes. There is a 1.5-cm mass in the right adrenal gland. A biopsy specimen of the lung mass is most likely to show which of the following? (A) B-cell lymphoma (B) Lung abscess (C) Mesothelioma (D) Metastatic adenocarcinoma of the breast (E) Multiple endocrine neoplasia (F) Non-small cell lung carcinoma (G) Sarcoidosis (H) Tuberculosis

Answer F is correct. The most common cause of a lung mass with surrounding lymphadenopathy in a long-term smoker is non-small cell lung carcinoma. Cough, shortness of breath, and weight loss are common findings. Metastasis to an adrenal gland or to supraclavicular lymph nodes would be consistent with advanced disease.

A previously healthy 42-year-old man is brought to the emergency department 1 day after the sudden onset of shortness of breath and chest pain at rest; the pain is exacerbated by deep inspiration. His pulse is 100/min, respirations are 22/min, and blood pressure is 140/90 mm Hg. Breath sounds are normal. The remainder of the examination shows no abnormalities. Arterial blood gas analysis on room air shows: pH 7.49 PCO2 30 mm Hg PO2 64 mm Hg O2 saturation 91% An x-ray of the chest shows no abnormalities. Which of the following is the most likely mechanism of these findings? (A) Carbon monoxide poisoning (B) Congenital heart disease (C) Depressed alveolar ventilation (D) Interstitial edema (E) Interstitial fibrosis (F) Low oxygen-carrying capacity of the blood (G) Ventilation-perfusion mismatch

Answer G is correct. Sudden onset shortness of breath, with pleuritic chest pain, tachycardia, hypoxemia, and low pCO2 suggest pulmonary embolism, which results in ventilation-perfusion mismatch. Answer A is incorrect. Carbon monoxide poisoning is a common and frequently unsuspected cause of inhalational injury and results in tissue hypoxia by competitively displacing oxygen from hemoglobin. Affinity of carbon monoxide for hemoglobin is about 250 times greater than that of oxygen. The correlation between carbon monoxide levels and symptoms is weak, but generally patients with levels greater than 30% are symptomatic. Symptoms may range from confusion or fatigue to nausea, headache, and profound coma. The diagnosis is based on clinical grounds and supported by laboratory data. Carbon monoxide intoxication might occur in closed automobiles and by exposure to kerosene heaters or charcoal fires in closed spaces. In suggested cases, arterial blood gas levels should be obtained with measured (not calculated) hemoglobin-oxygen saturation. A carbon monoxide level should be measured in patients with a measured systemic arterial oxygen saturation (SaO2) lower than the calculated SaO2 obtained from the arterial oxygen tension. Treatment includes 100% inspired oxygen. Answer B is incorrect. Congenital heart disease with right-to-left shunt is associated with hypoxemia. In this case, the heart examination is normal. Answer C is incorrect. Depressed alveolar ventilation is associated with an increased PCO2, not a decrease PCO2. Answer D is incorrect. Interstitial edema produces crackles on lung examination and abnormalities on chest x-ray. Answer E is incorrect. Interstitial fibrosis produces crackles on lung examination and abnormalities on chest x-ray. Answer F is incorrect. Anemia diminishes the oxygen-carrying capacity of the blood. There is no evidence of anemia is this patient.

A 3-year-old girl is brought to the physician because of a 3-day history of dark urine and a 1-day history of yellow skin and pale stools. One week ago, she had mild abdominal pain, vomiting, and diarrhea that have now resolved. Several other children at the day care she attends had similar symptoms 2 weeks ago. She has received her complete hepatitis B immunization series. She is at the 50th percentile for height and weight. Examination shows pale yellow skin and mild scleral icterus. There is mild tenderness of the liver to palpation. What is the most likely mechanism of jaundice? (A) Autoimmune disorder (B) Bacterial infection (C) Metabolic process (D) Obstruction (E) Physiologic process (F) Toxin exposure (G) Viral infection

Answer G is correct. The differential diagnosis for jaundice is large. The presence of abdominal pain, vomiting, and diarrhea that occurred in other children around the same time suggests a transmissible agent, likely a virus, causing hepatitis. Tenderness over the liver is typical of hepatitis.

A 5-year-old boy is brought to the physician because of a 2-day history of fever and painful swelling of the left ankle. He has had recurrent cervical lymphadenitis and pneumonia since infancy. Two years ago, a culture from an abscess in the cervical region grew Staphylococcus aureus. His temperature is 38°C (100.4°F). Examination shows a tender, erythematous, edematous left ankle; there is point tenderness over the medial malleolus. A bone scan shows increased uptake in the left lower tibial metaphysis. Culture of bone aspirate grows Serratia marcescens. Nitroblue tetrazolium test shows no color change. Which of the following is the most likely mechanism for these findings? (A) Adenosine deaminase deficiency (B) Consumption deficiency (C) Defective opsonization (D) Destruction of CD4+ T lymphocytes (E) Developmental arrest of maturation of B lymphocytes (F) Dysmorphogenesis of the third and fourth pharyngeal pouches (G) Impaired chemotaxis (H) Impaired phagocytic oxidative metabolism

Answer H is correct. Neutrophils from patients with chronic granulomatous disease (CGD) cannot develop an oxidative burst. Patients with chronic granulomatous disease are predisposed to disease by catalase-producing organisms such as staphylococci, Serratia, Nocardia, and Aspergillus. The nitroblue tetrazolium (NBT) test is a test for chronic granulomatous disease. It is a method for evaluating neutrophil oxidative burst dysfunction in CGD patients. Neutrophils ingest the dye, nitroblue tetrazolium, and in the presence of reactive oxygen species, the yellow colored NBT compound is converted to the purple-blue formazan compound. Neutrophils from patients with chronic granulomatous disease generate reactive oxygen species and therefore, cannot reduce the yellow NBT dye and thus there is no change in the color produced. Answer A is incorrect. Adenosine deaminase deficiency occurs in Severe Combined Immunodeficiency Disease. Answer B is incorrect. Complement activation occurs via either the classical or the alternative pathway, which converge at the level of C3 and share a sequence of terminal components. Four aspects of the complement cascade are critical to its function and regulation: (i) activation of the classical pathway, (ii) activation of the alternative pathway, (iii) C3 convertase formation and C3 deposition, and (iv) membrane attack complex assembly and insertion. In general, mechanisms evolved by pathogenic microbes to resist the effects of complement are targeted to these four steps. Although complement undoubtedly plays a role in host defense against many microbial pathogens, it appears most important in protection against encapsulated bacteria, especially Neisseria meningitidis but also Streptococcus pneumoniae, Haemophilus influenzae, and, to a lesser extent, Neisseria gonorrhoeae. Answer C is incorrect. Opsonization is diminished in sickle cell anemia. Answer D is incorrect. HIV (human immunodeficiency virus) destroys CD4+ T lymphocytes. Answer E is incorrect. Bruton's agammaglobulinemia is caused by a defect in Bruton's tyrosine kinase, an enzyme essential to B-cell, but not T-cell, development. Answer F is incorrect. DiGeorge syndrome is a genetic disorder caused by microdeletions of the q arm of chromosome 22 that result in dysmorphogenesis of the third and fourth pharyngeal pouches. This can result in thymic hypoplasia, decreased T-cell numbers/function and recurrent viral and fungal infections. Answer G is incorrect. Job's syndrome (hyperimmunoglobulin E syndrome) is a group of disorders characterized by abnormal neutrophil chemotaxis.

A 32-year-old woman comes to the physician because of an 8-month history of moderately severe, bilateral, diffuse headaches that last "all day, every day." They are not responsive to over-the-counter medications. She has not had nausea, vomiting, or aura. She appears mildly depressed. Neurologic examination shows no abnormalities. Which of the following is the most likely diagnosis? A. Increased intracranial pressure B. Migraine C. Occipital neuralgia D. Post-traumatic headache E. Subarachnoid hemorrhage F. Subdural hematoma G. Temporomandibular joint syndrome H. Tension-type headache

Answer H is correct. Steady diffuse headaches in the absence of nausea, vomiting, aura, and neurological findings are likely to be tension-type headaches. Answer A is incorrect. Headaches due to increased intracranial pressure are usually mild and worse in the morning or with exertion. Visual disturbances may occur and papilledema may be present. Answer B is incorrect. The pain of migraine is often pulsating, unilateral, and frontotemporal in distribution and often accompanied by anorexia, nausea, and, occasionally, vomiting. In characteristic attacks, patients are markedly intolerant of light (photophobia) and seek rest in a dark room. Some migraines are accompanied by aura. Answer C is incorrect. Occipital neuralgia is characterized by a sharp, lancinating pain lasting a few seconds and occurring repetitively in the occipital region. Symptoms may be provoked by neck extension. Answer D is incorrect. Post-traumatic headache occurs after head injury and is associated with irritability, concentration impairment, insomnia, memory disturbance, and light-headedness. Anxiety and depression are present to variable degrees. Answer E is incorrect. Subarachnoid headache is usually extremely severe and often described by the patient as "the worst headache of my life." Nuchal rigidity, third nerve palsy (usually involving the pupil), and retinal, preretinal, or subconjunctival hemorrhages may be found. Answer F is incorrect. Subdural hematoma occurs after head injury and is commonly associated with abnormalities on neurological exam. Answer G is incorrect. Temporomandibular joint (TMJ) syndrome is associated with pain and tenderness over the TMJ and muscles of mastication, as well as decreased range of motion of the TMJ.

A 2500-g (5-lb 9-oz) full-term newborn has opacities obscuring the fundi and a continuous murmur heard over the left hemithorax. He was born to a 21-year-old primigravid woman following an uncomplicated labor and delivery. The mother received no prenatal care, and pregnancy was complicated by an erythematous rash during the first trimester. She has had five sexual partners during the past year. Which of the following is the most likely pathogen? (A) Clostridium tetani (B) Cytomegalovirus (C) Escherichia coli (D) Group B streptococcus (E) Herpes simplex virus (F) Listeria monocytogenes (G) Parvovirus B19 (H) Rubella virus (I) Toxoplasma gondii (J) Treponema pallidum

Answer H is correct. The newborn has cataracts and patent ductus arteriosus, suggestive of congenital rubella syndrome. The mother's rash during the first trimester is consistent with rubella. Answer A is incorrect. Neonatal tetanus typically manifests within 3-12 days of birth as progressive difficulty in feeding (sucking and swallowing), associated hunger, and crying. Paralysis or diminished movement, stiffness and rigidity to the touch, hyperreflexia, and spasms, with or without opisthotonos, are characteristic. Because tetanus toxin does not affect sensory nerves or cortical function, the patient remains conscious. Answer B is incorrect. Cytomegalovirus (CMV) is the most common cause of in utero infections; however, only 10% of infants born with congenital CMV are symptomatic at birth. These more severely affected infants are more likely to have been exposed to a primary maternal infection during the first trimester. Most infants who are symptomatic at birth (hepatosplenomegaly, jaundice, petechiae and microcephaly) develop late complications such as deafness, intellectual disability, or seizures. Answer C is incorrect. The most common bacterial causes of neonatal meningitis are Group B Streptococcus, E. coli, and L. monocytogenes Answer D is incorrect. The most common bacterial causes of neonatal meningitis are Group B Streptococcus, E. coli, and L. monocytogenes. Answer E is incorrect. Herpes simplex infections can result in a triad of symptoms: skin vesicles or scarring; eye disease (chorioretinitis along with other eye findings such as keratoconjunctivitis), and microcephaly. The most common route of infection occurs during delivery (intrapartum). Infection with HSV in the infant is almost invariably symptomatic and frequently lethal. Infants who are infected intrapartum and postnatally can be divided into three 3 different catagories: disease localized to the skin, eye, or mouth; encephalitis with or without skin, eye, or mouth involvement; disseminated infection that involves multiple organs, including the CNS, lung, liver, adrenals, skin, eye, and /or mouth. Answer F is incorrect. The most common bacterial causes of neonatal meningitis are Group B Streptococcus, E. coli, and L. monocytogenes Answer G is incorrect. Erythrovirus (Parvovirus) B19 is a common viral infection of children that can result in erythema infectiosum (slapped cheek syndrome; fifth disease). The virus infects and lyses human erythroblasts. Perinatal and intrapartum infections are very rare. Infections in utero can result in fetal death (rare), nonimmune fetal hydrops (uncommon), birth defects (eyes, CNS), and prematurity. Answer I is incorrect. Toxoplasmosis is mainly acquired from cat feces or eating undercooked meat. The infection may be asymptomatic or produce only mild symptoms in the mother. Infection early in the pregnancy may cause the death of the fetus and abortion; infection later can cause fetal damage, stillbirth or a liveborn infant with damage to the brain and body organs. Symptomatic babies are born with hydrocephalus, chorioretinitis and cerebral calcification. Answer J is incorrect. Congenital syphilis is associated with fever, irritability, deformity of the nasal bridge (saddle nose), and a rash affecting the palms and soles. Newborns may have a rash around the mouth, genitalia, and anus that eventually scars.

A 67-year-old man is brought to the emergency department because of headache and decreased level of consciousness over the past hour. He has a 20-year history of poorly controlled hypertension. He is unresponsive to verbal stimuli. With painful stimulation, he moves the left extremities semipurposely but not the right extremities. The left pupil is larger than the right and reacts sluggishly to light. Babinski sign is present on the right. A CT scan of the head shows a large hyperintense mass in the left basal ganglia with compression of the left lateral ventricle and shift of the midline structures. Which of the following is the most likely diagnosis? (A) Acute cerebral infarction (B) Arteriovenous malformation (C) Astrocytoma (D) Bacterial abscess (E) Carotid cavernous fistula (F) Epidural abscess (G) Epidural hematoma (H) Fungal abscess (I) Glioblastoma multiforme (J) Intracerebral hemorrhage (K) Medulloblastoma (L) Meningioma (M) Metastatic carcinoma (N) Mycobacterium avium-intracellulare granuloma (O) Sturge-Weber syndrome (P) Subarachnoid hemorrhage (Q) Subdural hematoma

Answer J is correct. Headache and decreased level of consciousness in a 67-year-old man with hypertension and a hyperintense mass on CT indicates intracerebral hemorrhage. In this vignette, there is evidence of left uncal herniation. (The uncus squeezes the third cranial nerve, affecting the parasympathetic input to the eye, which causes the pupil to dilate and become less reactive to light.)

Which of the following statements is FALSE regarding opioid therapy? A. Nonpharmacologic therapy is preferred to opioid therapy for treating chronic pain B. Nonopioid pharmacologic therapy is preferred to opioid therapy for treating chronic pain C. If opioids are used for chronic pain, they should be combined with nonpharmacologic therapy as appropriate D. If opioids are used for chronic pain, they should be combined with nonopioid pharmacologic therapy as appropriate E. When starting opioid therapy for chronic pain, clinicians should prescribe immediate-release opioids instead of extended-release/long-acting (ER/LA) opioids F. For most patients with severe acute pain requiring opioids, a 10-day prescription of an immediate-release opioid with no refills is appropriate G. Urine drug testing is recommended at least yearly for patients receiving opioids for chronic pain H. Clinicians should avoid prescribing opioid pain medication and benzodiazepines concurrently whenever possible

The correct answer is F. Inappropriate use of opioids to treat acute pain often leads to chronic opioid use and its complications. Physicians often overestimate the duration of opioid therapy needed to treat acute pain. When opioids are used for severe acute pain, clinicians should prescribe the lowest effective dose of immediate-release opioids and should prescribe no greater quantity than needed for the expected duration of pain severe enough to require opioids. Three days or less will often be sufficient; more than seven days will rarely be needed. More than a few days of opioid therapy significantly increases the risk of addiction. Each day of unnecessary opioid use increases likelihood of physical dependence. Answer A is incorrect. Nonpharmacologic therapy, such as cognitive behavioral therapy and exercise therapy, may be beneficial in patients with chronic pain. Given the potential complications of chronic opioid therapy (e.g., addiction, dependence, drug tolerance), nonpharmacologic therapy is preferred to opioid therapy in responsive patients. Answer B is incorrect. Nonopioid pharmacologic therapy, such as acetaminophen, NSAIDs, cyclooxygenase 2 (COX-2) inhibitors, selected anticonvulsants, and selected antidepressants (particularly tricyclics and serotonin and norepinephrine reuptake inhibitors [SNRIs]), may be beneficial in patients with chronic pain. Given the potential complications of chronic opioid therapy (e.g., addiction, dependence, drug tolerance), nonopioid pharmacologic therapy is preferred to opioid therapy in responsive patients. Answer C is incorrect. If opioids are used to treat chronic pain, they should be combined with nonpharmacologic therapy and nonopioid pharmacologic therapy as appropriate to provide greater benefits to patients in improving pain and function. Answer D is incorrect. If opioids are used to treat chronic pain, they should be combined with nonpharmacologic therapy and nonopioid pharmacologic therapy as appropriate to provide greater benefits to patients in improving pain and function. Answer E is incorrect. There is a higher risk for overdose among patients initiating treatment with extended release/long-acting opioids than among those initiating treatment with immediate-release opioids. Answer G is incorrect. Urine drug tests can provide information about drug use that is not reported by the patient. In addition, urine drug tests can assist clinicians in identifying when patients are not taking opioids prescribed for them, which might in some cases indicate diversion. (Note: urine drug tests do not provide accurate information about how much or what dose of opioids a patient took.) Urine drug testing is recommended at least yearly for patients receiving opioids for chronic pain. Answer H is incorrect. Benzodiazepines and opioids both cause central nervous system depression and can decrease respiratory drive. Concurrent use is likely to put patients at greater risk for potentially fatal overdose.


Conjuntos de estudio relacionados

Florida Statutes, Rules, and Regulations

View Set

Life Insurance Settlement Options

View Set

Taxes, retirement, and other insurance concepts

View Set

Chapter 10: The Psychiatric-Mental Health Nursing Process

View Set

Online Expierence Stack 1 ism 280

View Set

Texas principles of real estate - part 1

View Set